#2 Rosh Review

Ace your homework & exams now with Quizwiz!

Question: What mnemonic is used to remember the clinical symptoms of hypercalcemia?

"Stones, bones, abdominal moans, and psychic groans."

Question: In the U.S., what is the prevalence of COPD?

15 million. Rapid Review Cor Pulmonale Pulmonary HTN + RVH → right heart failure MC chronic cause: COPD MC acute cause: PE Right heart catheterization

Question: What is the most common cause of testicular torsion?

A congenital malformation known as a "bell-clapper" deformity. Less than 10% of cases have a traumatic etiology. Rapid Review Testicular Torsion Bimodal: < 1-year-old, puberty Increased risk: undescended testicle, Bell-Clapper deformity Sudden onset of unilateral pain during sleep or exercise Left > right Absent cremasteric reflex Doppler ultrasound Immediate urologic consultation Manual detorsion (medial to lateral/opening a book) if delays expected

Question: What percent of body calcium is found in the skeleton?

Answer: 98%.

Question: What is intersection syndrome?

Answer: An overuse tendinopathy manifesting with pain to the radial side of the wrist 4-8 cm proximal to the site of de Quervain's disease. Rapid Review de Quervain Tenosynovitis Patient will be complaining of radial wrist pain PE will show pain on ulnar wrist deviation with thumb in fist (Finkelstein's test) Treatment is thumb spica splint, NSAIDs

Question: What is the most common cause of acute coronary syndrome?

Answer: Atherosclerosis. Rapid Review Acute Coronary Syndrome: Management Aspirin: ↓ mortality, ↓ infarct size, ↓ reinfarction rate Clopridogrel: patients with aspirin allergy Heparin: ↓ DVT, ↓ reinfarction, ↓ stroke, ↓ LV thrombus, ↓ reocclusion Nitroglycerin: Coronary artery dilation/vascular smooth muscle relaxation → ↓preload/afterload → ↓ myocardial O2 demand Contraindications: sildenafil use within 24 hrs, RV infarction ß-blockers: ↓ Myocardial O2 demand, ↓ ventricular fibrillation IV indications: tachydysrhythmias, intractable HTN Morphine: ↓ Preload/afterload, ↓ sympathetic activity No mortality benefit Glycoprotein IIb/IIIa inhibitors: benefit in patients undergoing PCI PCI: Preferred over thrombolytics in all STEMI patients PCI center: <90 minutes contact to device time Non-PCI center: transfer to PCI center if contact to device time can be <120 minutes Non-PCI center: thrombolytics if contact to device time to be >120 minutes Thrombolytics: begin within 30 minutes of ED arrival if selected

Question: What is the disorder called which patients deliberately impose harm on self for the principal purpose of assuming the sick role?

Answer: Factitious disorder (previously referred to as Munchausen syndrome).

Question: Children who have been treated for bacterial meningitis should undergo which kind of evaluation?

Answer: Hearing test.

Question: What is the cause of condyloma acuminatum?

Answer: Human papillomavirus.

Question: How does mitral stenosis result in hoarseness?

Answer: Left atrial enlargement can cause compression of the recurrent laryngeal nerve resulting in hoarseness.

Question: What infectious etiology is associated with complete heart block?

Answer: Lyme disease. Rapid Review Heart Block: Second Degree Type I (Wenckebach/Mobitz I) Progressive PR interval prolongation until QRS dropped Block within AV node Usually benign

Question: What is the treatment for oral hairy leukoplakia?

Answer: Most cases are self-limited, but antiviral agents like acyclovir, ganciclovir and foscarnet can be considered. Rapid Review Oral Hairy Leukoplakia EBV Lateral aspect of tongue Lesions do not scrape off (unlike thrush) HIV

Question: What additional management is indicated in a bite from a dog with unknown immunization status?

Answer: On initial presentation, the patient should have rabies immunoglobulin administered and should have the rabies vaccination series started. Rapid Review Cat and Dog Bites Pasteurella multocida (most common pathogen in cat wounds) Abx for all cat wounds (amoxicillin-clavulanate) Most cat wounds left open Sepsis or gangrenous wound after dog bite = Capnocytophaga canimorsus

Question: Where does Stensen's duct drain?

Answer: Opposite the upper second molar.

Question: Infection with what common childhood virus can cause a transient aplastic crisis in patients with sickle cell disease?

Answer: Parvovirus B19.

Question: What is the carotid sinus reflex?

Answer: Pressure applied to the carotid sinus enhances the baroreceptor firing rate and activates vagal efferents, thereby slowing the heart rate and reducing blood pressure.

Question: What organism is responsible for hot tub folliculitis in most cases?

Answer: Pseudomonas aeruginosa.

Question: What distinguishes chickenpox from smallpox?

Answer: The crops of smallpox all develop with the same timing whereas in chickenpox they present in various stages.

Question: Is the rotavirus vaccine contraindicated for an infant with a history of intussusception?

Answer: Yes.

Question: What is the most commonly associated complication of mitral valve stenosis?

Atrial fibrillation. Rapid Review Mitral Stenosis Most common cause: rheumatic heart disease Dyspnea on exertion, hemoptysis Opening snap, diastolic apical murmur Atrial fibrillation

Question: What is quadruple therapy for peptic ulcer disease?

Bismuth subsalicylate, metronidazole, tetracycline, and a PPI. Rapid Review Upper GI Bleed Bleeding proximal to the ligament of Treitz Most common cause: PUD Melena NGT aspirate: blood Endoscopy

Question: What is the most common fatality in refeeding syndrome?

Cardiac complication.

Question: How does carotid massage affect 2nd degree type I and type II heart block?

Carotid massage will typically worsen 2nd degree type I block and improve 2nd degree type II block.

Question: What is a complication of an ocular chemical burn injury?

Corneal abrasion.

A man presents to the emergency department with a blood pressure of 200/136 mm Hg and laboratory evidence of acute renal failure. Which of the following medications and initial blood pressure response rates are the most appropriate at this time? Intravenous labetalol, to goal within 1 to 2 hours Intravenous nicardipine, to goal within 6 to 12 hours Oral clonidine, to a goal within 3 to 6 days Oral magnesium sulfate, to a goal within 1 to 2 days

Correct Answer ( A ) Explanation: A hypertensive emergency is a severe elevation in blood pressure with evidence of end-organ damage. This requires immediate lowering of blood pressure. In hypertensive emergencies, treatment begins in the emergency department and continues in the intensive care unit. It begins with obtaining intravenous access for medication delivery, obtaining intra-arterial access for monitoring and serial measurements of urinary output, blood pressures, mental status, and renal indices. Medication options include labetalol, esmolol, nicardipine, hydralazine, fenoldopam, nitroglycerin, nitroprusside, and phentolamine. IV labetalol (a beta-adrenergic and alpha-adrenergic blocker) has a rapid onset of action (within minutes), making it very useful for the treatment of hypertensive emergencies. In most hypertensive emergencies, the mean arterial pressure (MAP) should be reduced by about 10-20% in the first hour, followed by a gradual reduction during the next 23 hours. The treatment goal is to achieve a final pressure reduction by 25% compared with baseline. In hypertensive emergencies, the goal of treatment is to decrease the mean arterial pressure by 25% over minutes to 2 hours, with an ultimate goal of obtaining diastolic stability < 110 mm HG within 2 to 6 hours (B). Acute exacerbation of chronic hypertension is treated with oral medications to a goal of normal blood pressure over 1-2 days (C). Hypertensive crisis during pregnancy is likely part of eclampsia, a seizure-hypertension syndrome that occurs in 12% of pregnancies. Initial treatment includes volume expansion, intravenous antihypertensives (namely labetalol or nicardipine), seizure prophylaxis with magnesium sulfate (D) and immediate delivery.

A teenage girl presents with an eyelid lump that began 3 months ago. It has noticeably enlarged over the past 2 weeks. On inspection, you appreciate a mass which seems to be originating on the internal surface of the superior right eyelid. Palpation of this mass reveals a non-greasy, non-tender, non-mobile nodule. Attempted expression of this nodule offers no discharge. Which of the following is the most likely diagnosis? Chalazion Hordeolum Nevus Seborrheic keratosis

Correct Answer ( A ) Explanation: A slow growing, chronic subcutaneous eyelid nodule is called a chalazion. These nodules are typically non-tender, and can occur within the lid, or on the internal or external surface of the lid. Obstruction of the meibomian gland is the most common cause, leading to formation of a lipogranulomatous mass, which is usually non-tender. Chalazia may or may not become secondarily infected, and they may or may not be accompanied by blepharitis (generalized eyelid inflammation). Treatment consists of lid hygiene, topical antibiotics and ultimately, intralesional steroids. A hordeolum (B), or stye, is a more acutely presenting, painful eyelid lesion. It more commonly occurs on the lid margin, but can also occur on the internal surface. Internal surface eyelid lesions typically develop from plugging of a meibomian gland, while lid-margin or external surface eyelid lesions may represent infection of the more anteriorly located glands of Moll/Zeiss. Hordeola usually self-drain within one week of lid hygiene and antibiotic ointment treatment. Nevi (C) are well-demarcated, usually flat or minimally elevated, pigmented or nonpigmented lesions that are present at birth. This patient's lesion began 3 months ago. Seborrheic keratosis (D) is a pigmented, greasy, hyperkeratotic lesion that appears to be stuck on the skin. They are more common in the elderly population.

What is the most common cause of minor hemoptysis in the Emergency Department? Acute bronchitis Pneumonia Pulmonary embolism Tuberculosis

Correct Answer ( A ) Explanation: Acute bronchitis is the most common cause of minor hemoptysis in the Emergency Department. Hemoptysis describes the presence of blood in sputum expectorated from the respiratory tract. The majority of episodes of hemoptysis are minor episodes with small amounts of blood. Less than 5% of patients with hemoptysis have life-threatening hemorrhage. Bronchitis results from inflammation in the bronchial tree and can lead to hemoptysis. Additionally, forceful coughing further irritates the inflamed mucosal surfaces and can result in bleeding. Hemoptysis in bronchitis is typically self-limited. Pneumonia (B), pulmonary embolism (C) and tuberculosis (D) can all cause hemoptysis as well but are less common causes than acute bronchitis.

A seven-year-old boy is seen in your clinic with cough, wheezing, abdominal pain, and diarrhea. He recently returned from a family vacation to India. You get stool studies that yield a diagnosis of ascariasis. Which of the following is the best medication for this child? Albendazole Ivermectin Metronidazole Pyrantel pamoate

Correct Answer ( A ) Explanation: Ascariasis (Ascaris lumbricoides round worm) infection is a result of ingestion of infective eggs. Initially, children may present with cough, dyspnea, wheezing, and hemopytsis as a result of larvae travel via systemic circulation to the lungs. After lung invasion, the larvae travel down small intestines by passing over the epiglottis and are subsequently swallowed. This can lead to abdominal pain, distension, small bowel obstruction, peritonitis, and malabsorption. Stool eggs can be detected approximately six to eight weeks after ingestion and often symptoms do not present till weeks after ingestion. The treatment of choice is a one-time oral dose of albendazole 400 mg. This medication is metabolized after absorption and distributes systemically, making it very effective. Ivermectin (B) is the drug of choice in the treatment of Strongyloidiasis. Metronidazole (C) is not indicated in the treatment of helminth infections. Pyrantel pamoate (D) can be used as second-line agent for the treatment of ascariasis. The regimen is 11mg/kg orally for three days.

A patient is being discharged from the hospital after having a ST-elevation myocardial infarction. During his stay, he underwent percutaneous coronary intervention with placement of a drug eluting stent. The patient is being sent home on the following medications: aspirin 81 mg, metoprolol 50 mg, nitroglycerin 0.4 mg sublingual, and atorvastatin 40 mg. Which of the following should also be added to his regimen? Clopidogrel Fish oil Ranolazine Reteplase

Correct Answer ( A ) Explanation: Clopidogrel (Plavix®) is a platelet P2Y12 receptor blocker. Similar agents include ticagrelor (Brillenta®) and prasugrel (Effient®). Dual antiplatelet therapy with aspirin and a platelet P2Y12 receptor blocker (such as clopidogrel) decreases the risk of coronary artery stent thrombosis and its consequences of myocardial infarction or death more than the use of aspirin alone. The duration of therapy of dual antiplatelet therapy differs for each type of stent. The current recommendation is treatment with a platelet P2Y12 receptor blocker for at least one year after placement of a drug eluting stent and one month after bare metal stent. Aspirin should be continued indefinitely for all stented patients. Fish oil (B) is sometimes recommended for primary and secondary prevention of coronary artery disease but is not essential for post percutaneous coronary intervention therapy. Ranolazine (C) is an anti-anginal medication that is added only after calcium channel blockers, beta-blockers, and nitrates have failed to control angina. Reteplase (D) is a fibrinolytic agent used to treat ST-elevation myocardial infarctions when a catheterization lab is not readily available. None of these agents play a role in preventing stent thrombosis.

Which of the following is the most common cause of cor pulmonale? Chronic obstructive pulmonary disease Connective tissue disorders Left-sided cardiac disease Right ventricular hypertrophy

Correct Answer ( A ) Explanation: Cor pulmonale is a condition of altered right heart function due to respiratory disease, and is estimated to account for 7% of all types of adult heart disease in the U.S. Acute forms are due to pulmonary emboli and acute respiratory distress syndrome. On the other hand, cor pulmonale more commonly has a chronic progressive course. Some pathophysiologic mechanisms include pulmonary vasoconstriction, pulmonary vascular bed compromise due to pulmonary interstitial or alveolar disease, connective tissue disease and high blood viscosity. There are several underlying etiologies, the most common being COPD (>50%). Others include pulmonary hypertension, rheumatoid disease, pulmonary thromboemboli, polycythemia vera, and sickle cell disease. Although connective tissue disease (B) can result in pulmonary vascular compromise and subsequent cor pulmonale, it is by far not the most common cause. A key fact in the diagnosis of cor pulmonale is that it is not due to left-sided heart disease (C). Right ventricular hypertrophy (D) is the major result of, not cause of, cor pulmonale.

A 52-year-old man presents with a red right eye that is associated with severe pain and blurred vision. On exam there is some cloudiness to the cornea. Pupils are 5 mm OD and 3 mm OS. Which of the following is the most likely diagnosis? Acute glaucoma Conjunctivitis Iritis Keratitis

Correct Answer ( A ) Explanation: Glaucoma is characterized by an increase in intraocular pressure leading to irreversible damage to the optic nerve. There are two forms of glaucoma: open-angle and angle-closure. Open-angle glaucoma is the most common form of glaucoma and is most often seen in the elderly population. Acute angle-closure glaucoma is an ophthalmologic emergency and is caused by closure of the anterior chamber. Symptoms are rapid onset and usually unilateral. It presents with severe pain and profound vision loss. As intraocular pressure increases, patients may experience headache, nausea, and vomiting. Exam reveals a red, tender globe that may be firm to touch. A steamy or hazy cornea may be seen with a nonreactive, dilated pupil. Treatment should be initiated to reduce the intraocular pressure. Patients require emergent ophthalmologic consultation. Conjunctivitis (B) is not associated with severe pain or a dilated pupil. Iritis (C) is typically associated with ciliary flush, blurred vision, and a clear cornea. The pupil in keratitis (D) is reactive and not dilated.

Which of the following patients with a renal stone should be considered for admission? A 24-year-old man with a 5 mm stone at the ureterovesicular junction who presents with intractable nausea and vomiting A 33-year-old pregnant woman with a 2 mm stone at the ureteropelvic junction who is tolerating oral intake and has 2+ hematuria A 37-year-old man with a 3 mm stone at the pelvic brim; ultrasound shows no hydronephrosis; urinalysis shows 2+ blood and 5-10 WBCs A 45-year-old man with a 7 mm renal stone at the ureterovesicular junction who required a single dose of pain medication to resolve his pain

Correct Answer ( A ) Explanation: Most patients with renal stones may be safely discharged home from the ED. The patient in the above scenario, with intractable nausea and vomiting, should be admitted to the hospital for symptomatic treatment relief and fluid resuscitation. Sepsis and renal damage are risks in the presence of obstruction and infection. These patients require immediate urologic consultation to evaluate the need for drainage and for relief of the obstruction by ureteral stenting. If infection is present, patients require fluid resuscitation and antibiotics. A 33-year-old pregnant woman with a 2 mm stone at the ureteropelvic junction who is tolerating oral intake and has 2+ hematuria (B) can be discharged, but will require close follow-up. There should be a low threshold to admit pregnant women with renal stones, particularly when there are signs of infection or dehydration. A 37-year-old man with a 3 mm stone at the pelvic brim, where ultrasound shows no hydronephrosis and urinalysis shows 2+ blood and 5-10 WBCs (C) can be discharged. His stone is likely to pass spontaneously given its size. Although he has WBCs in his urine, there is no evidence of obstruction seen by the lack of hydronephrosis on ultrasound. A 45-year-old man with a 7 mm renal stone at the ureterovesicular junction who required a single dose of pain medication to resolve his pain (D) can be discharged with outpatient follow-up to a urologist. His stone is less likely to pass spontaneously. Therefore, he may require lithotripsy. This can be done outpatient.

You examine a two-year-old boy for a routine health visit. He eats a healthy diet. He can speak six words. He can also point to one body part. He can walk up the steps. You give him a crayon, and he can scribble. His length is at the 45th percentile and weight is the 30th percentile. Physical examination is normal. The family recently acquired a puppy, which has not yet received vaccinations. You discuss rabies with the family. Which of the following is the best statement regarding rabies? All bites from animals that can be infected with rabies virus carry a risk of transmission Post-exposure prophylaxis should be given within a week if an exposing animal is suspected to be rabid Rabies cases among domestic animals have increased in the United States The risk of acquisition of rabies does not increase with multiple bites

Correct Answer ( A ) Explanation: Rabies is a fatal viral disease primarily acquired from the bite of a rabid animal. All bites from animals that can be infected with rabies virus carry a risk of rabies transmission. The risk of acquisition increases with multiple bites, and with bites in closer proximity to the central nervous system (CNS). Children, presumably because of their size are more at risk as well. Bites by certain animals, particularly bats, may be too small to be routinely detected or may occur at night during sleep. Canine rabies was responsible for most human deaths in the United States prior to the 1950s, and was largely eliminated as a source by the 1940s. In the late 1970s, wild animals emerged as the source of the now rarer cases of human disease in the United States. During 2008, rabies animal surveillance in the United States identified the four major animal reservoirs as raccoons, bats, skunks, and foxes. Post-exposure prophylaxis should be given within a week if an exposing animal is suspected to be rabid (B) is false since post-exposure prophylaxis should be given immediately without any delay. Rabies cases among domestic animals have increased in the United States (C) is wrong because rabies cases have declined. The risk of acquisition of rabies does not increase with multiple bites (D) is false because risk of acquisition increases with multiple bites.

A 12-year-old girl with cystic fibrosis has been non-compliant with her vitamin and enzyme supplements. She presents with dry eyes and difficulty seeing at night. She is most likely deficient in which vitamin? Vitamin A Vitamin D Vitamin E Vitamin K

Correct Answer ( A ) Explanation: Signs of vitamin A deficiency includes dry conjunctiva, Bitot spots on the conjunctiva (keratin debris), night blindness, and ultimately permanent blindness. Vitamin A is necessary for the formation of the photoreceptor rhodopsin in the retina and it helps maintain the epithelial tissues. Precursors of vitamin A are found in yellow and green leafy vegetables and bright-colored fruits. Vitamin A deficiency is a leading cause of blindness in developing countries, where the diet is deficient in this nutrient. This is more common in southern and eastern Asia where people rely on rice, which is deficient in Beta-carotene. Secondary vitamin A deficiency can occur from absorption problems such as cystic fibrosis, pancreatic insufficiency, chronic diarrhea, giardiasis and cirrhosis. Vitamin A can also shorten the duration of measles and decrease the severity of symptoms. Vitamin A levels are measured by serum retinol. Like Vitamin A, Vitamins D, E and K are fat-soluble vitamins that can be deficient in patients with cystic fibrosis. Vitamin D deficiency (B) can lead to rickets, osteomalacia, and muscle aches and weakness. Vitamin E deficiency (C) causes impairments in the immune response, neurological problems, ataxia and anemia due to oxidative damage to red blood cells. Vitamin K deficiency (D) affects coagulation factors II, VII, IX, X and protein C and protein S which leads to prolonged prothrombin time and increased bruising, petechiae and hematomas.

A 75-year-old otherwise healthy woman states that she has passed out three times in the last month during her daily brisk walk. Which one of the following is the most likely cause of her syncope? Aortic stenosis Atrial myxoma Orthostatic hypotension Vasovagal syncope

Correct Answer ( A ) Explanation: Syncope with exercise is a manifestation of organic heart disease in which cardiac output is fixed and does not rise with exertion. Syncope, commonly occurring with exertion, is reported in up to 42% of patients with severe aortic stenosis. The pathology of aortic stenosis includes processes similar to those in atherosclerosis, including lipid accumulation, inflammation, and calcification. The development of significant aortic stenosis tends to occur earlier in those with congenital bicuspid aortic valves. During the asymptomatic latent period, left ventricular hypertrophy and atrial enlargement of preload compensate for the increase in afterload caused by aortic stenosis. As the disease worsens, these compensatory mechanisms become fail, leading to symptoms of heart failure, angina, or syncope. Doppler echocardiography is the recommended initial test for patients with classic symptoms of aortic stenosis. It is helpful for estimating aortic valve area, peak and mean transvalvular gradients, and maximum aortic velocity. Aortic valve replacement should be recommended in most patients with any of these symptoms accompanied by evidence of significant aortic stenosis on echocardiography. Vasovagal syncope (D) is associated with unpleasant stimuli or physiologic conditions, including sights, sounds, smells, sudden pain, sustained upright posture, heat, hunger, and acute blood loss. Orthostatic hypotension (C) is associated with changing from a sitting or lying position to an upright position. Atrial myxoma (B) is associated with syncope related to changes in position, such as bending, lying down from a seated position, or turning over in bed.

A 32-year-old man presents with a left testicular mass. Which of the following examination characteristics is most suggest a primary testicular cancer? Nontender and firm Nontender and fluctuant Tender and soft Tender and transilluminates

Correct Answer ( A ) Explanation: Testicular cancer can occur at any age, but most commonly is seen at ages 20-35 years. Risk factors include cryptorchidism, family history, infertility, tobacco use, and exposure to DES in utero. The clinical presentation may be asymptomatic or present with a painless mass in the scrotum. Patients may complain of a dull ache or pain, swelling, or hardness in the lower abdomen or scrotum. Exam reveals a firm, hard, or fixed nodular mass on the testis that does not transilluminate. All patients with a testicular mass need to be evaluated for testicular cancer. Workup should include testicular ultrasound and serum tumor markers: beta-hCG, LDH, and alpha-fetoprotein. Cancerous testicular masses are not typically fluctuant (B). Fluctuance usually denotes inflammation or fluid, as in epididymitis, hematocele or hydrocele. Cancerous testicular masses typically are nontender (C and D) and do not transilluminate (D).

A 32-year-old man who works as a carpenter presents with leg numbness. On physical exam, he has decreased light touch and pinprick sensation only in an area around his front pants pocket. Distal leg sensation, strength and reflexes are intact. Which of the following nerves is most likely involved? Lateral femoral cutaneous Pudendal Saphenous Sural

Correct Answer ( A ) Explanation: The lateral femoral cutaneous nerve is a direct branch of the lumbosacral plexus, whose main contribution is from the L2 and L3 roots. It travels inferiorly, running lateral to the femoral nerve and underneath the inguinal ligament, ultimately sending cutaneous branches to the proximal, mainly lateral, thigh. This is a common entrapment site for the lateral femoral cutaneous nerve, especially in patients with large abdomens, as in pregnancy or obesity. It also can also be irritated by repetitive trunk flexion in a patient who wears large belts, as in a carpenter whose tool belt compresses the inguinal region during repetitive forward bending. The pudendal nerves (B) innervate the pelvic floor musculature, and transmit sensation from the perineal region. The saphenous nerve (C), a distal branch of the femoral nerve, is a pure sensory nerve that transmits sensation from the medial lower leg, medial ankle and arch of the foot. The sural nerve (D), a distal branch of the tibial and common peroneal nerves, is also a pure sensory nerve, and transmits sensation from the posterolateral lower leg, lateral ankle and lateral foot.

A 2-year-old boy presents with a burn to his right hand as seen above. The patient's mother states that he mistakenly got burned by hot water when she tipped a hot tea kettle over. What management is indicated? Consultation with child services for suspected abuse Consultation with plastic surgery for skin grafting Discharge home with silvadene and follow up Transfer to a burn center

Correct Answer ( A ) Explanation: This patient presents with a partial thickness second degree burn with a mechanism of injury that does not fit the injury pattern raising suspicion for child abuse. The most likely etiology of these burns is from a cigarette. Cigarette burns are typically round and sharply-demarcated. They are sometimes confused with healing impetigo. Child physical abuse refers to infliction of injury to any part of the child. This may present as bruising, fractures, brain injury, burns or internal hemorrhage. Often patients will present with multiple injuries in various stages of healing or patterned injuries (resembling objects). Burns may occur from contact with a hot object or with immersion in hot water. Although accidental hot water burns are common, those sustained from abuse will have characteristic patterns as well. Immersion injuries to extremities will present with glove-stocking distribution involvement. Additionally, intentional immersion injuries may present with burns to the anogenital area. Children with burn injuries with these patterns should always be investigated for possible abuse. Small, isolated burns rarely need intervention from a plastic surgeon for skin grafting (B) or transfer to a burn center (D). The patient should not be discharged home (C) until a full evaluation has been made into the possibility of child abuse.

A 25-year-old woman with a recent history of antibiotic use presents to your office with a complaint of vaginal discomfort. For the past week she has been experiencing intense vaginal pruritus and has noticed a white discharge. Which of the following is the most appropriate next step in her management? A single dose of 150 mg fluconazole Boric acid intravagina for seven days Metronidazole 500 mg twice/day for seven days Topical nystatin 100,000 units daily for seven days

Correct Answer ( A ) Explanation: Vulvar or vaginal pruritus is the most common clinical symptom of vulvovaginal candidiasis. Risk factors include recent antibiotic use, poor glycemic control in women with diabetes mellitus, immunosuppression, increased estrogen levels and use of certain contraceptive devices such as diaphragms. Vaginal discharge is not always seen in women with vulvovaginal candidiasis, however when present it is generally thick and white with a clumpy, cottage cheese-like texture. First line treatment is a single dose of fluconazole, 150 mg. Boric acid (B) and topical nystatin (D) are used in women who have complicated vulvovaginal candidiasis and who are allergic to fluconazole. Complicated infections are defined as having severe signs or symptoms, the presence of a candida species other than Candida albicans, or in women with a history of recurrent vulvovaginal candidiasis infections, poorly controlled diabetes mellitus, pregnancy, or immunosuppression. Metronidazole (C) is used in the treatment of bacterial vaginosis.

An 18-year-old woman presents with a laceration to her face from a dog bite that occurred 10 hours ago. The patient owns the dog. Examination reveals a 4 cm laceration to the left cheek with no signs of infection. What is the most likely management that is indicated? Irrigation and antibiotics Irrigation and primary wound closure Primary wound closure after irrigation and antibiotics Primary wound closure and antibiotics

Correct Answer ( B ) Explanation: Mammal bites to any part of the body should be copiously irrigated and explored followed by an assessment for primary closure. In this patient, primary closure is recommended as the laceration is on the face. Canine bites often involve laceration as well as crush injury to tissue depending on the size of dog. The presence of a crush injury may make primary wound repair difficult. Additionally, devascularization of the tissue may make primary closure contraindicated as the risk of infection increases. Classically, it was taught that lacerations sustained from dog bites should be irrigated, given antibiotics and not primarily repaired because of these risks. *However, more recent literature has shown that the risk of infection was no different for primary closure versus healing by secondary intention. Additionally, if the laceration is to a cosmetic area like the face, primary repair should be attempted.* As with any laceration, tetanus status should be updated. Copious irrigation and wound exploration is central to good wound care. Exploration should pay particular attention to the presence of foreign bodies especially teeth, which may break off during the bite. *Antibiotics (A & C) are not routinely needed for dog bites despite classic teaching. Antibiotics should be reserved for patients with signs of infection, multiple comorbidities or large wounds with gross contamination. If antibiotics are given, they should primarily cover Staphylococcus and Streptococcus species, as these are the predominant organisms in the canine oral cavity.* Eikenella and Pasturella are less commonly responsible for infections. Irrigation and antibiotics alone (A) would be indicated for dog bites that are grossly infected or have large defects that cannot be primarily closed. Wound closure and antibiotics without irrigation (D) is also contraindicated as copious irrigation is central to proper wound management.

A patient is found to have a low pitched rumbling diastolic apical murmur. Which of the following is the most frequent presenting complaint associated with this murmur? Chest pain Dyspnea with exertion Hemoptysis Palpitations

Correct Answer ( B ) Explanation: Mitral stenosis is characterized by left ventricular inflow obstruction resulting in a low pitched rumbling diastolic apical murmur. Rheumatic heart disease is the most common cause worldwide. Valvular obstruction develops slowly over time, taking decades to become symptomatic. Less common causes of mitral stenosis include congenital mitral stenosis and mitral annular calcification. Mitral valve obstruction causes elevated left atrial pressures resulting in left atrial enlargement and pulmonary hypertension. Dyspnea with exertion is the most common presenting complaint, found in up to 70% of patients. It is exacerbated by factors that increase cardiac demand (eg. fever, anemia, pregnancy). As the degree of stenosis increases, less exertion is necessary to produce symptoms and orthopnea can occur. Chest pain (A) is an uncommon complaint in mitral stenosis. Hemoptysis (C) due to bronchial vein rupture can occur in mitral stenosis but is less common. Palpitations (D) can occur secondary to atrial fibrillation which is a common complication of mitral stenosis. The prevalence of atrial fibrillation increases with higher degrees of stenosis, advanced age and the presence of other valvular abnormalities. Patients with atrial fibrillation require rate control and anticoagulation.

You are caring for a patient with rheumatoid arthritis. A rheumatologist refills this patient's medications, but does not order the proper laboratory testing used to monitor for adverse side effects. You decide to begin such monitoring. Which of the following is most commonly serially monitored in patients on disease-modifying antirheumatic drugs (DMARDs)? Chest radiograph Complete blood count Rheumatoid factor Thyroid function test

Correct Answer ( B ) Explanation: Rheumatoid arthritis (RA) is treated with a variety of medications, from salicylates to disease-modifying anti-rheumatic drugs (DMARDs). Several, if not all, of these medications carry significant side effects and toxicities. As such, routine monitoring of potential side effects or pathology is recommended and necessary. Common side effects of these medications include nausea, vomiting, diarrhea and rash. However, one of the most common DMARD adverse events is bone marrow suppression, especially with sulfasalazine, methotrexate, gold preparations, leflunomide, cyclosporine and azathioprine. Therefore, it is important to monitor marrow health with serial complete blood counts. Hepatitis and hepatic toxicity can occur with chronic methotrexate and leflunomide. As such, clinicians should obtain liver function tests at baseline and periodically throughout treatment. Leflunomide and cyclosporine are associated with hypertension. As such, routine monitoring of blood pressure is important in patients on these medications. Gold preparations are associated with significant kidney damage, namely proteinuria. It is recommended to check serial urinalyses, even on a weekly basis, in patients taking gold sodium thiomalate. Checking a PPD test for tuberculosis is also recommended. Repeated chest X-rays (A) are not routinely recommended in monitoring the effects of DMARD medications. Rheumatoid factor (C) is not a very sensitive or specific marker of rheumatoid arthritis. However, a screen for this antibody is routinely performed, but commonly negative (especially in early disease), to help secure a diagnosis of RA, not monitor for DMARD side effects. Eventually, nearly 70-80% of patients with RA will have a positive RF. DMARDs are not commonly associated with thyroid disease (D).

An 18-month-old girl presents to the Emergency Department with a rash. Mom reports that she has had a high fever for the last three days. Today, the fever broke, but she developed a maculopapular rash on her trunk as seen in the picture above. What is the likely diagnosis? Erythema infectiosum Roseola Rubeola Scarlet fever

Correct Answer ( B ) Explanation: Roseola infantum is caused by human herpesvirus 6 and the vast majority of cases are seen in children under two years old. The disease is characterized by a high fever (often as high as 41 oC) for 3-5 days, which ends abruptly and is followed by the onset of a diffuse fine maculopapular rash on the trunk which may spread to the face and extremities. There is no mucous membrane involvement. Children affected are typically not ill appearing. The illness is self-limited, with the rash lasting only a few days. Treatment consists of fever control as needed. Erythema infectiosum (A), caused by parvovirus B19, is a nonfebrile illness that affects children ages 4-10 years old. Children will have a bright erythematous rash on the cheeks ("slapped cheek" appearance) followed by a lacy rash on the trunk. Rubeola (C) is characterized by cough, coryza, conjunctivitis, and fever followed by a maculopapular rash on the head which spreads to the body. Patients with rubeola are typically still febrile when the rash appears. Scarlet fever (D) presents with fever, malaise, and sore throat followed by a fine sandpaper-like rash.

A seven-year-old boy is brought by his mother to the clinic because of coughing. For the past week, he has had a nonproductive cough. On physical examination, vital signs are normal, with erythematous posterior pharynx, and clear breath sounds. Complete blood count is normal. Chest radiograph reveals perihilar infiltrates. Polymerase chain reaction from the boy's nasopharyngeal specimen comes back positive for Mycoplasma pneumoniae. Which of the following is the most appropriate therapy? Amoxicillin Clarithromycin Levofloxacin Oseltamivir

Correct Answer ( B ) Explanation: The boy has a lower respiratory tract infection due to Mycoplasma pneumoniae. The cough caused by M. pneumoniae infection ranges from nonproductive to mildly productive. Wheezing and dyspnea also may occur, although dyspnea is not a common complaint. Chills are common, but rigors are very rare. Additional respiratory symptoms include pharyngitis, rhinorrhea, and ear pain. Compared to those with pyogenic pneumonia, patients with mycoplasma pneumonia tend to have a more gradual onset of symptoms, less respiratory distress, and usually a normal white blood cell count. The Infectious Diseases Society of America (IDSA) suggests serology or polymerase chain reaction (PCR) tests for the laboratory diagnosis of M. pneumoniae. When available, PCR from a nasopharyngeal specimen can be done rapidly, has a high specificity, and is the diagnostic test of choice. Empiric treatment for M. pneumoniae pneumonia often is initiated based on clinical suspicion given the difficulty with definitive diagnosis. Suspected or documented M. pneumoniae lower respiratory tract infections are treated with a macrolide or tetracycline antibiotic, which is consistent with those in the 2011 Pediatric Infectious Diseases Society (PIDS) and the Infectious Diseases Society of America (IDSA) guidelines. Among the macrolide antibiotics, clarithromycin and azithromycin have a more convenient dosing schedule and fewer side effects than erythromycin, but erythromycin is less expensive. High-dose amoxicillin (A) can be given for the majority of bacterial pathogens causing community-acquired pneumonia. However, it is not preferred for atypical causes of pneumonia such as M. pneumoniae. Levofloxacin (C) is a fluoroquinolone which is not a first-line therapy for pediatric patients. Oseltamivir (D) is given for infection due to influenza.

A mother brings her 9-year-old daughter to the ED because she is complaining of nighttime itching in the anal area ever since returning from summer camp one week ago. What is the appropriate treatment for this condition? Clotrimazole cream Mebendazole Mupirocin ointment Sitz baths

Correct Answer ( B ) Explanation: The patient has enterobiasis (pinworms). The treatment is with single-dose mebendazole (or albendazole), an oral anthelmintic. This infection is most commonly seen in children between the ages of five and fourteen. The condition is caused by ingesting eggs of Enterobius vermicularis. The eggs hatch within the intestinal tract, and female parasites lay new eggs in the perianal area, leading to itching (pruritus ani) that is characteristically worse at night. Secondary skin changes and bacterial infection may also be present. Other associated problems may include insomnia, enuresis, UTIs, and vaginitis. All family members should be treated. In households where more than one member is infected or where repeated, symptomatic infections occur, it is recommended that all household members be treated at the same time. Clotrimazole cream (A) is indicated for candidal infections, which are possible in the perianal area and can also lead to itching. However, this patient's presentation is more consistent with enterobiasis. Likewise, mupirocin ointment (C) is used for gram-positive bacterial skin infections such as impetigo. Sitz baths (D) are used for painful perineal conditions such as anal fissures, hemorrhoids, and healing episiotomies. They work by keeping the affected area clean and promoting blood flow. In the case of enterobiasis, cleansing is important, but an anthelmintic agent is necessary to eradicate the infection.

A 20-year-old man who is employed in construction presents with pain to his thumb and wrist for 4 months. He states that the pain worsens while he is working. Examination reveals pain along the radial surface of the wrist with forced ulnar abduction. What management is indicated? MRI of the wrist NSAIDs and thumb spica splint Surgical release of the dorsal extensor compartment Wrist X-ray with ulnar deviation view

Correct Answer ( B ) Explanation: The patient presents with de Quervain's disease, or tenosynovitis, and should be managed conservatively with splinting and NSAIDs. De Quervain's disease is a repetitive strain injury that is often work related. In this disorder, the abductor pollicus longus (APL) and extensor pollicus brevis (EPB) tendons found in the dorsal extensor compartment of the wrist are inflamed and enlarged. Patients report pain on the radial surface of the wrist, which is increased with movement of the thumb or ulnar deviation at the wrist. Patients may have decreased grip strength as well. Finkelstein's test is considered to be pathognomonic for de Quervain's disease. It involves ulnar deviation of the wrist with the thumb in full flexion. This movement creates severe pain over the styloid of the radius. First line treatment is with rest, splinting (thumb spica splint) and NSAIDs. MRI of the wrist (A) is not necessary in the diagnosis of de Quervain's disease but is useful if a scaphoid fracture is suspected. Surgical release of the dorsal extensor compartment (C) is indicated for patients with refractory symptoms. A wrist X-ray with ulnar deviation (D) can reveal a small scaphoid fracture in trauma to the wrist.

A 52-year-old man presents from his primary care physician's office for evaluation of an abnormal electrocardiogram. His ECG is seen above. He has no symptoms. What is the appropriate intervention? Measurement of cardiac enzymes No intervention Telemetry observation Transcutaneous pacer pad placement

Correct Answer ( B ) Explanation: This ECG demonstrates an example of Type 1 second-degree atrioventricular (AV) block. AV block results from impaired conduction through some portion of the electrical circuit impairing communication between the atria and ventricles. In first and second degree AV block there is a partial disruption of the electrical circuitry as opposed to third degree heart block when there is no electrical communication between the atria and ventricles (AV dissociation). In second-degree heart block, some sinus impulses do not reach the ventricles at all. In type 1 second-degree AV block, there is lengthening of the PR interval until ultimately a beat is dropped. This is also known as Wenckebach or Mobitz I. In many cases, type 1 second-degree heart block is a normal variant. Other times, the etiology is likely related to increased vagal tone and in most cases requires no treatment. It can also occur in a myocardial infarction and usually resolves after the infarct period. Type 2 second-degree heart block is characterized by a dropped beat without any prolongation of the PR interval. Type 2 is never considered a normal variant. In the absence of any cardiac symptoms, the measurement of cardiac enzymes (A) in type 1 second-degree heart block is not necessary. Telemetry observation (C) is not required in type 1 second-degree heart block because it does not typically progress into other forms of heart block. In patients with type 2 heart block, this interruption of the electrical pathway occurs below the level of the AV node and can progress. Transcutaneous pacer pads (D) are often placed on the chest of a patient with complete heart block (type 3) because of the high chance for impaired cardiac output. These patients will require pacemaker placement if the heart block is not due to a reversible cause (e.g. hyperkalemia).

A 34-year-old woman presents to the ED with chest pain that is worse with inspiration and better upon leaning forward. She has had a runny nose and cough for the last week. In the ED, her vital signs are BP 134/78, HR 86, RR 14, oxygen saturation 99% on room air, and T101°F. On exam, a friction rub is heard. An ECG displays global ST segment elevation with PR segment depression. What is the most likely diagnosis, and what would be the next step in management? Acute myocardial infarction; give aspirin, nitroglycerin, consult cardiology, and activate the cath lab Acute pericarditis; give nonsteroidal anti-inflammatory drugs Cardiac tamponade; perform immediate pericardiocentesis Pulmonary embolism; order CT angiography of the chest

Correct Answer ( B ) Explanation: This patient most likely has acute pericarditis, which is inflammation of the pericardial sac. Patients present with pleuritic chest pain that is typically worse when lying supine, deep inspiration, or swallowing. The pain is usually relieved by leaning forward. On auscultation, a pericardial friction rub may be heard. Pulsus paradoxus may also be observed, which is a fall in systolic blood pressure of greater than 10 mmHg with inspiration. Pericarditis can have many etiologies including infection, systemic connective tissue diseases, uremia, post-radiation, or post-myocardial infarction (Dressler's syndrome). Although there is no definitive diagnostic test, an ECG can demonstrate diffuse ST segment elevation, diffuse PR segment depression, and PR elevation in aVR (thumbprint sign). Treatment of pericarditis is mainly supportive. NSAIDs will reduce inflammation and pain. Steroids or colchicine may be given for refractory cases. An acute myocardial infarction (A) is less likely in this patient given her age and clinical presentation (fever, runny nose, and cough). A pericardial effusion may accompany acute pericarditis but rarely leads to cardiac tamponade (C). The clinical triad of pericardial tamponade is hypotension, jugular venous distension, and muffled heart sounds. A pulmonary embolism (D) is less likely in this patient given her ECG findings, vital signs, and clinical presentation.

Which of the following symptoms is the most common presentation of urinary bladder cancer? Colicky abdominopelvic pain and hematochezia Gross, painless hematuria Lumbosacral pain Nocturia, dysuria and urinary hesitancy

Correct Answer ( B ) Explanation: Urinary bladder cancer is the 2nd most common genitourinary cancer and affects men more than women. The mean age at diagnosis is 65 years. Risk factors include cigarette smoking (60%), exposure to industrial dyes or solvents (15%), arsenic exposure, chronic cystitis, schistosomiasis, radiation, and cyclophosphamide use. Transitional cell carcinoma is most common. Hematuria is the most common presenting symptom. Irritative or obstructive voiding symptoms may also be seen. Advanced disease may present with flank, pelvic or bony pain or lower extremity edema. In any patient over age 40 who presents with gross hematuria, bladder cancer needs to be ruled out. Workup includes cystourethrosopy, urine cytology, and CT with urography or IVP with renal ultrasound. Treatment options range from resection of visible tumor for superficial cancer to radical cystectomy and chemotherapy. Colicky lower abdominopelvic pain and hematochezia (blood thru the anus, in or with stool) (A) are common presenting symptoms of colorectal cancer, not bladder cancer. An advanced, but typically not presenting, symptom of bladder cancer could be lumbosacral pain (C). Nocturia, dysuria and urinary hesitancy (D) are common presenting symptoms of prostate cancer.

A 44-year-old woman with multiple sclerosis presents with 3 weeks of urine leakage. She does not have a strong desire to urinate prior to her most distressing complaint of "dribbling". She has no history of pelvic surgery. She denies dribbling after coughing or sneezing. Which of the following best describes this patient's urinary incontinence? Deformity (lack-of-continuity) Overflow Stress Urge

Correct Answer ( B ) Explanation: Urinary incontinence is a common disorder in primary care. It is more common in women than men. The key to the diagnosis and successful treatment is understanding the different categories of incontinence. Overflow incontinence is characterized by unpredictable dribbling of urine or weak urine stream due to detrusor hyporeflexia (underactive bladder) or urinary outlet obstruction. Underactive bladder may be due to medications (e.g. calcium channel blockers, anticholinergics) or detrusor denervation or injury. Outlet obstruction may be due to enlarged prostate gland, tumors, urethral stricture, or chronic constipation. Treatment includes discontinuation of medications that promote urinary retention. Prazosin and terazosin are alpha-blockers that induce internal sphincter relaxation and may relieve retention associated with benign prostatic hyperplasia (BPH). 5-alpha-reductase inhibitors such as finasteride may partially relieve symptoms associated with BPH, but require months for onset of effect. Acute urinary retention and acontractile bladders require indwelling or intermittent catheterization. Deformity incontinence (A), also known as lack-of-continuity incontinence, is due to fistula formation, which may occur in patients with prior hysterectomy or other pelvic surgeries. Stress incontinence (C) is characterized by an involuntary loss of urine during coughing, laughing or exercise. Its pathology is based on weak bladder neck support and urethral hypermobility. Urge incontinence (D), sometimes called overactive bladder, is the involuntary loss of urine preceded by a strong urge to urinate even though the bladder may not be full. Associated symptoms include urgency, frequency, nocturia and dysuria. Causes include detrusor hyperreflexia, detrusor instability, chronic UTIs, bladder cancer and cystolithiasis.

A patient presents with the lesion seen above on his upper lid. The lesion developed over weeks. It is nontender. This condition is associated with blockage of which gland?

Correct Answer ( C ) Explanation: A chalazion is a lipogranuloma of either a meibomian gland or a Zeis gland. When the meibomian gland is involved, the lid nodule is characteristically hard and painless. If the Zeis gland is involved, it is marginal or superficial. Lipid breakdown products, possibly from bacterial enzymes or from retained sebaceous secretions, leak into the surrounding tissue and incite a granulomatous inflammatory response. The resulting mass of granulation tissue and chronic inflammation (with lymphocytes and lipid-laden macrophages) distinguishes a chalazion from an internal or external hordeolum (stye) which is primarily an acute pyogenic inflammation with polymorphonuclear leucocytes and necrosis with pustule formation. Upon examination, the single, nontender, firm nodule or, in rare cases, multiple nodules are located deep within the lid or the tarsal plate, whereas a hordeolum (stye) is more superficial and is typically centered on an eyelash. Eversion of the lid may reveal the dilated meibomian gland and chronic inspissation of adjoining glands. With judicious pressure on the lid, the thick secretions can be seen extruding like toothpaste, resulting in tear debris. Chalazia are more common on the upper lid than on the lower lid because of the increased number and length of meibomian glands present on the upper lid. Small, inconspicuous, asymptomatic chalazia may be ignored. Conservative treatment with lid massage, moist heat, and topical mild steroid drops usually suffices. Rarely one may administer oral tetracycline (eg, doxycycline 100 mg or minocycline 50 mg every day for 10 days) to minimize the infectious component and decrease the inflammation. Chronic therapy with low-dose tetracycline (eg, doxycycline 100 mg orally every week for 6 months) frequently prevents recurrence. The ciliary (A) gland, also known as the gland of Moll, is located at the base of the eyelashes and is associated with external hordeolum. The lacrimal glands (B) are situated in the upper, outer portion of each orbit. This gland produces tears. Obstruction of this gland leads to dacrocystadenitis. The pineal gland (D) is located in the brain and produces melatonin which helps regulate the sleep cycle.

An 18-month-old girl presents to urgent care with profuse mucoid nasal discharge and cough. She has had nasal discharge for the past 2 weeks with no improvement from using a humidifier. She has also had fever for the past four days, with a Tmax of 103°F. She has not been able to attend daycare for the past week due to the fever and persistent symptoms. Which of the following is the best initial therapy for this child? Azithromycin Ceftriaxone High dose amoxicillin-clavulanate (80-90 mg/kg/day of amoxicillin) Low dose amoxicillin (45 mg/kg/day)

Correct Answer ( C ) Explanation: Acute sinusitis is a common illness of childhood, characterized by fever, cough, purulent nasal discharge, and nasal congestion. The most common cause of sinusitis is viral, which is best treated with supportive care. Acute bacterial sinusitis often follows a case of viral sinusitis. In young children, sinusitis may be present in the ethmoidal sinuses. The maxillary sinuses are present at birth, but are not pneumatized until 4 years of age. The sphenoid sinuses are present by age 5, and the frontal sinuses begin development at age 7-8. Due to this child's persistent symptoms for more than 10-14 days, fever of greater than 102°F, and purulent nasal discharge for more than 3 consecutive days, the most likely diagnosis is acute bacterial sinusitis. The most common bacterial pathogens are Streptococcus pneumoniae (30%), nontypable Haemophilus influenzae (20%), and Moraxella catarrhalis (20%). Less common causes include other strains of streptococci, Staphylococcus aureus, and anaerobic bacteria. Initial treatment consists of low dose amoxicillin, which covers the most common bacterial pathogens. However, some children are at risk for resistant strains of bacterial pathogens, such as children in daycare, those less than 2 years of age, and those who have received antibiotics in the preceding 1-3 months. These children should be given amoxicillin-clavulanate with high dose amoxicillin. Children who fail initial therapy should also be escalated to high dose amoxicillin-clavulanate. Azithromycin (A) is an alternative antibiotic that can be used to treat sinusitis in older children. It would not be the first line therapy in this young child. Ceftriaxone (B) should be used in frontal sinusitis, complicated sinusitis (such as periorbital or orbital cellulitis) or in the setting of intracranial complications (such as epidural abscess, meningitis, or cavernous sinus thrombosis). Low dose amoxicillin (D) is the first line therapy in uncomplicated sinusitis, when the child does not have risk factors for resistant bacterial pathogens.

What is the most frequently isolated organism in acute suppurative parotitis? Bacteroides sp. Escherichia coli Staphylococcus aureus Streptococcus viridans

Correct Answer ( C ) Explanation: Acute suppurative parotitis, which occurs when decreased salivation or stasis results in migration of oral bacteria to the gland, presents with facial erythema, swelling, and tenderness over the parotid gland. Patients are typically elderly with a history of recent surgery or hospitalization, dehydration, or anticholinergic medication use. Although polymicrobial in etiology, Staphylococcus aureus is the most frequently isolated pathogen. Other organisms may include S. viridans, gram negative bacteria, such as E. Coli and Eikenella, and anaerobes such as Bacteroides. Pseudomonas may be present in hospitalized patients. Although Bacteroides (A), E. Coli (B) and S. viridans (D) may be present in acute suppurative parotitis, none are not the most frequently isolated pathogen.

Which of the following physical exam findings is most suggestive of appendicitis? Kehr's sign Murphy's sign Rovsing's sign Scarf sign

Correct Answer ( C ) Explanation: Appendicitis is an inflammation of the vermiform appendix, which is located at the base of the cecum near the ileocecal valve. Appendicitis occurs when the lumen of the appendix is obstructed, generally by lymphoid hyperplasia, fecaliths, infections or parasites. It most commonly affects individuals in the second and third decades of life. Symptoms of appendicitis include abdominal pain, anorexia, nausea and vomiting. The abdominal pain generally begins as periumbilical pain that migrates to the right lower quadrant. Fever may occur later in the course of the illness. A number of physical exam signs may be elicited to help facilitate diagnosis. These include Rovsing's sign, which occurs when palpation of the left lower quadrant elicits pain in the right lower quadrant. Diagnosis is based on history and physical exam and is confirmed by imaging, either computed tomography (CT) or ultrasound. Treatment is surgical intervention and removal of the appendix. Kehr's sign (A) is referred pain to the left shoulder that worsens with inspiration in patients with splenic injuries. Murphy's sign (B) is elicited in the physical exam for suspected cholecystitis by instructing the patient to take a deep inspiration while palpating the right costal margin just beneath the edge of the liver. A positive Murphy's sign occurs when the patient experiences discomfort and catches their breath on the inhalation. Scarf sign (D) is used in the motor function evaluation of a newborn to determine range of shoulder adduction.

You suspect hypoparathyroidism in a patient who recently underwent a course of neck radiotherapy. Which of the following laboratory results would you expect to find if this patient began showing signs of carpopedal spasm and lip paresthesias? Hypercalcemia and hyperphosphatemia Hypercalcemia and hypophosphatemia Hypocalcemia and hyperphosphatemia Hypocalcemia and hypophosphatemia

Correct Answer ( C ) Explanation: Calcium and phosphate homeostasis is maintained by an interplay between the parathyroid glands, intestines, kidneys and bones. The normal action of parathyroid hormone (PTH) is to increase serum calcium levels and decrease serum phosphate levels. PTH affects the kidneys by increasing calcium reabsorption, decreasing renal phosphate reabsorption, and increasing active-vitamin D formation, and affects the bones by increasing osteoclast activity. Hypoparathyroidism can have primary and secondary causes. In primary hypoparathyroidism, the causes are congenital, acquired (such as thyroidectomy, parathyroidectomy, neck radiation), autoimmune or associated with magnesium deficiency or heavy metal diseases. Patients may present with symptoms of hypocalcemia: paresthesias (especially of the lips, mouth and fingertips), cramping, tetany, carpopedal spasm, altered mental status, irritability, mood swings, anxiety, stridor and hoarseness, wheezing and bronchospasm, diaphoresis and seizures. Laboratory testing which suggests primary hypoparathyroidism includes decreased PTH, hypocalcemia and hyperphosphatemia. Hypercalcemia and hyperphosphatemia (A), hypercalcemia and hypophosphatemia (B), and hypocalcemia and hypophosphatemia (D) are not associated with hypoparathyroidism

Which of the following is the immediate first step in the treatment of chemical injuries to the eye? Application of topical antibiotics to prevent infection Complete ocular examination, including dilated fundoscopic examination Manual removal of particulate material followed immediately by irrigation with saline until the pH is 7.0 Topical steroids and artificial tears

Correct Answer ( C ) Explanation: Chemical injuries to the eye represent one of the true ophthalmic emergencies. While almost any chemical can cause ocular irritation, serious damage generally results from either strongly basic (alkaline) compounds or acidic compounds. Alkali injuries are more common and can be more deleterious. Bilateral chemical exposure is especially devastating, often resulting in complete visual disability. Immediate, prolonged irrigation, followed by aggressive early management and close long-term monitoring, is essential to promote ocular surface healing and to provide the best opportunity for visual rehabilitation. Immediate irrigation with normal saline is the initial step even before complete exam when a patient presents with a chemical eye injury. A lid speculum should be placed and topical anesthesia applied. Irrigation may be administered by a handheld bottle or through IV tubing with an irrigation lens. The pH should be checked with a pH strip and irrigation discontinued when the pH reaches 7.0. Any particulate matter should be removed prior to irrigation if it is a reactive substance such as ammonium hydroxide crystals since fluid may dissolve these causing more injury. The upper lid should be everted to check for any particulate matter. Once irrigation of the eye is complete, then a complete ocular examination may be indicated (B). Antibiotic ointments can serve the dual purpose of providing lubrication and preventing infection. Broad-spectrum antibiotic coverage (A) is required to most effectively minimize infection. However these are indicated after proper irrigation and eye exam have been completed. Artificial tears and ointments (D) are especially important with severely scarred and exposed eyes. Ascorbate, both oral and topical, aids in the synthesis of collagen fibrils. Topical steroids decrease ocular surface inflammation, facilitating new epithelial cell growth and ocular surface regeneration. However application of these is not considered the initial step in the management of ocular chemical injuries.

A previously healthy 49-year-old obese woman presents to your office with complaints of increased thirst, frequent urination, weight loss and blurry vision for the past several months. Which of the following is the most likely diagnosis? Diabetes insipidus Diabetes mellitus type 1 Diabetes mellitus type 2 Hypercalcemia

Correct Answer ( C ) Explanation: Diabetes mellitus type 2 is the most common type of diabetes among adults and occurs as a result of decreased insulin secretion and resistance to the action of insulin. Patients with diabetes mellitus type 2 are often asymptomatic and the diagnosis may be discovered as an incidental finding on screening. Risk factors for diabetes include being overweight, family history of diabetes mellitus, sedentary lifestyle, age greater than 45 years, history of gestational diabetes mellitus, hypertension, hyperlipidemia, and polycystic ovary syndrome. Risk factor identification helps determine which patients should be screened. Symptoms that point to a diagnosis of diabetes mellitus type 2 are due to hyperglycemia and include increased thirst, polyuria, nocturia, weight loss, and blurred vision. Diagnosis is established with elevated fasting plasma glucose, two-hour post oral glucose challenge or glycated hemoglobin (A1C) values on at least two occasions. First-line treatment for newly diagnosed diabetes involves lifestyle modifications and pharmacologic therapy with metformin. Diabetes insipidus (A) has two forms, central and nephrogenic and is characterized by the passage of large amounts of dilute urine. Diabetes insipidus is a rare, acquired disorder that can occur after trauma or surgery affecting the pituitary or hypothalamus. Diabetes mellitus type 1 (B) has similar symptoms to diabetes mellitus type 2, but more commonly presents in childhood. Patients with diabetes mellitus type 1 often present with diabetic ketoacidosis, which requires hospitalization due to the severity of symptoms. Patients with hypercalcemia (D) may present with urinary symptoms such as polyuria or nocturia. The majority of cases of hypercalcemia are due to hyperparathyroidism or malignancy.

A patient presents 2 weeks following a myocardial infarction. He complains of chest pain that improves with leaning forward, fever, and malaise. Vital signs are BP 125/70, HR 105, RR 14, and pulse oxygenation 98% on room air. Lab results reveal a leukocytosis and negative troponin. ECG shows sinus tachycardia. Which of the following is the most likely diagnosis? Acute myocardial infarction Bacterial pneumonia Dressler's syndrome Spontaneous pneumothorax

Correct Answer ( C ) Explanation: Dressler's syndrome or postcardiotomy pericarditis is due to an inflammatory reaction to transmural myocardial necrosis. It usually presents as a recurrence of pain with pleural-pericardial features. On physical exam, a friction rub may be heard. The troponin may be residually elevated from the initial myocardial infarction. It can occur from 2-10 weeks post-myocardial infarction. Treatment is with NSAIDs and steroids. The patient's ECG and troponin do not reveal any evidence of acute ischemia. Therefore it is unlikely the patient is experiencing an acute myocardial infarction (A). It is possible for the troponin to be residually elevated from the initial myocardial infarction. Bacterial pneumonia (B) is associated with fever and cough with yellow or rusty colored-sputum. Patients may experience pleuritic chest pain, but typically it is not affected by position. Spontaneous pneumothorax (D) is associated with decreased breath sounds on the side of the pneumothorax. It is not associated with fever or leukocytosis.

A 48-year-old man presents with fever and an acutely painful scrotum. He has significant pain during testicular palpation. A cremasteric reflex is present and Doppler ultrasonography shows an enlarged, thickened epididymis with increased blood flow to the left testicle. There is no discharge from the urinary meatus. Which of the following is the most appropriate antibiotic for this condition? Ceftriaxone plus doxycycline Doxycycline alone Levofloxacin Penicillin

Correct Answer ( C ) Explanation: Epididymitis occurs most commonly in men between the ages of 14 and 35 years. However, it can occur in any age group. It occurs from an ascending infection from the urethra, prostate, or bladder, and occasionally by hematogenous spread. Epididymitis is characterized by gradually increasing dull, unilateral scrotal pain, fever, and dysuria. Examination usually reveals localized epididymal edema and tenderness (posterior aspect of scrotum), possible testicular tenderness, and a normal cremasteric reflex. Pain may be relieved with testicular elevation (positive Prehn sign). Scrotal pain should be initially evaluated with a color Doppler ultrasound test, and in the case of epididymitis, the typical findings are an enlarged, thickened epididymis with increased blood flow. The most common organisms responsible for epididymitis in those 14 to 35 years-of-age are Neisseria gonorrhoeae and Chlamydia trachomatis. In older individuals (traditionally >35 years of age) and nonsexually active individuals, the Gram-negative rod bacteria (Escherichia, Klebsiella, Enterobacter and Citrobacter species) are most common. Trimethoprim-sulfamethoxazole or a fluoroquinolone such as levofloxacin or ciprofloxacin is the recommended treatment in this age group. Ceftriaxone plus doxycycline (A) is the treatment of choice for suspected orchitis or epididymitis in men between the ages of 14 and 35 years. Doxycycline alone (B) is not recommend as the sole antibiotic for orchitis in any age group. Penicillin (D) is more appropriate for streptococcal or staphylococcal infections, both of which are not common etiologies of orchitis or epididymitis.

Which of the following is the most common cause of an esophageal perforation? Acid ingestions Caustic ingestions Iatrogenic Mallory-Weiss syndrome

Correct Answer ( C ) Explanation: Most esophageal perforations are iatrogenic and often result from complications of instrumentation (about 60% of all cases). The rigid endoscope is the most common offender. Injuries tend to occur near the cricopharynx or the cervical esophagus as the endoscope is inserted. Acidic ingestions (A) typically lead to coagulative necrosis. Although this may lead to esophageal perforation, it is uncommon. Caustic ingestions (B) are bimodal in distribution and most commonly occur between the ages of one and five (accidental) and teens to their 20s (intentional). Ingested alkali (liquefaction necrosis) more commonly leads to esophageal perforation than acid does (coagulation necrosis). Mallory-Weiss syndrome (D) is most commonly due to violent and repeated vomiting or retching. It is associated with a partial thickness tear of the right posterolateral aspect of the distal esophagus or gastric cardia. Most cases are mild and resolve spontaneously.

In patients with a history of HIV infection and a CD4 count less than 200, which of the following opportunistic infections is trimethoprim-sulfamethoxazole (TMP-SMX) administered for prophylaxis? Coccidioidomycosis Disseminated mycobacterium avium complex disease Pneumocystis pneumonia Streptococcus pneumoniae

Correct Answer ( C ) Explanation: Pneumocystis jiroveci (formerly called Pneumocystis carinii) is an opportunistic pathogen in patients with HIV/AIDS. It causes a rapidly progressive fungal pneumonia characterized by severe hypoxia. In individuals with HIV infection, chemoprophylaxis with trimethoprim-sulfamethoxazole (TMP-SMX) is recommended when CD4 counts below 200 cells or there is a history of oropharyngeal candidiasis to help prevent PCP pneumonia. One double strength tablet daily is the recommended regimen, but one single strength tablet is also effective and better tolerated. Rapid Review Pneumocystis Pneumonia (PCP) Immunodeficiency HIV patients: CD4<200 CXR: bat wing pattern LDH: prognostic Rx: TMP-SMX Indications for steroids: children, PO2< 70 mm Hg, A-a gradient > 35 Chemoprophylaxis for coccidioidomycosis (A) is not recommended. Patients should receive prophylaxis with azithromycin and clarithromycin for disseminated mycobacterium avium complex (MAC) disease (B) if their CD4 count falls below 50. Both the pneumococcal (D) and influenza vaccines should be given to all patients with HIV, regardless of their CD4 count.

What disease is suggested by an increased hemoglobin concentration, leukocytosis, and thrombocytosis? Chronic myelogenous leukemia (CML) Multiple myeloma Polycythemia vera Pulmonary disease

Correct Answer ( C ) Explanation: Polycythemia can be divided into primary (polycythemia vera) and secondary. Polycythemia vera is a myeloproliferative disease in which all cell lines are increased and hemoglobin levels are typically greater than 18.5 g/dL in men and 16.5 g/dL in women. Symptoms are generally due to sludging of blood flow leading to poor oxygen delivery. Common complaints include weakness, headache, vision changes, and bleeding. Thrombosis and hemorrhage due to ischemia secondary to thrombosis are the main causes of death in patients with polycythemia vera. Secondary causes of polycythemia include conditions that increase erythropoietin (EPO) levels such as lung disease, heart disease, high altitude, and EPO secreting tumors. CML (A) is a myeloproliferative disease in which a neutrophilic leukocytosis predominates. In CML, the leukocyte alkaline phosphatase score is decreased (it is increased in polycythemia vera). Multiple myeloma (B) is a myeloproliferative disease in which plasma cells are overproduced and often "crowd out" other cell lines. Pulmonary disease (D) is a common cause of secondary polycythemia but would not be expected to cause an increase in all cell lines.

A 22-year-old college student complains of severe testicular pain upon waking this morning. Upon presentation two hours later, his left testicle is swollen and the pain begins to radiate toward his abdomen. Elevation of the testicle worsens the pain. There is no spermatic cord swelling and the cremasteric reflex is absent. Which of the following is the most likely diagnosis? Hydrocele Indirect inguinal hernia Torsion Varicocele

Correct Answer ( C ) Explanation: Testicular torsion occurs when the spermatic cord twists, cutting off arterial supply to the testicle resulting in ischemia. It is most common in infants, teens, and men <30 years of age . The clinical presentation includes an acute onset of testicular pain and swelling. The pain may radiate to the inguinal and hypogastric areas. Associated symptoms include nausea and vomiting. Examination reveals an extremely tender and swollen testicle. Pain may worsen when the testicle is elevated. The cremasteric reflex is absent. Immediate urologic consultation should be obtained when the diagnosis is suspected. Testicular torsion constitutes a urological emergency; cessation of blood supply for 4-6 hours can lead to permanent loss of the testicle. Treatment is surgical correction. Hydrocele (A) usually presents with a much more gradual onset of testicular swelling than torsion. Indirect inguinal hernia (B) also presents with abdominal pain, but in this condition, the testicle is usually normal, although swelling about the spermatic cord may be present. Varicocele (D) presents more commonly with painless swelling of the spermatic cord, not the testicle.

You examine a 17-year-old girl due to anorexia nervosa. She has low self-esteem with intense fear of gaining weight. She restricted her diet for the past three months. On physical exam, her BMI is 18 with heart rate of 35 beats per minute at rest. You decide to admit her for inpatient management. You plan to start nutritional therapy but worry about the risk of refeeding syndrome. Which laboratory finding is most consistent with refeeding syndrome? Hyperkalemia Hypermagnesemia Hypophosphatemia Thrombocytopenia

Correct Answer ( C ) Explanation: The core features of anorexia nervosa are restriction of energy intake, intense fear of gaining weight, and distorted perception of body weight. Patients with food-restricting subtype of anorexia nervosa should be evaluated for complications. Refeeding syndrome is a complication that occurs during nutritional therapy of malnourished patients. It is marked by hypophosphatemia, hypokalemia, thiamine deficiency, congestive heart failure and peripheral edema. Hypophosphatemia originates when phosphate stores are depleted during starvation. When nutritional replenishment begins and patients are fed with carbohydrates, glucose causes release of insulin that triggers cellular uptake of phosphate. Insulin also causes cells to produce adenosine triphosphate (ATP) and 2, 3-diphosphoglycerate (2, 3 DPG) that require phosphate, therefore further depleting phosphate stores. Complications of the syndrome may be reduced by slowing the rate of nutritional support and proactively correcting electrolyte abnormalities. Patients should be fed according to a standard protocol that includes a limited intake of sodium and fluids with daily calories raised by 300-400 kcal every three to four days. Hyperkalemia (A) and hypermagnesemia (B) are not expected findings. Refeeding syndrome leads to hypokalemia and hypomagnesemia. This is due to cellular uptake of phosphate, potassium, and magnesium once nutritional support is started. Thrombocytopenia (D) is not present in refeeding syndrome.

A 23-year-old man is admitted for a femur fracture. An admission ECG is shown above. What management is indicated? Atropine Beta blocker Reassurance Transvenous pacing

Correct Answer ( C ) Explanation: The patient's ECG shows a 1st degree AV block; a normal variant finding in 2% of the population that requires no specific management. 1st degree AV block is defined as prolonged conduction of atrial impulses. The PR interval > 0.20 seconds (200 msec) and there is no loss of any atrial impulse. All impulses result in a ventricular response. Although it can be a normal variant, it can be seen in pathologic conditions like Lyme disease and should be followed up. However, no specific management is needed. Nodal depressing agents should be avoided or used with caution. Rapid Review Heart Block: First-Degree Delayed AV node conduction PR interval > 0.2 seconds No specific rx Atropine (A) is required for patients with symptomatic bradycardia. Beta blockers (B) are AV nodal blockers and should be used with caution in patients with 1st degree AV blocks. Transvenous pacing (D) is required for higher degree AV blocks (2nd degree type II and 3rd degree heart block).

A 3-week-old baby girl with an unknown birth history presents to the emergency department with 3 days of poor feeding and recurrent cyanotic episodes. On exam, she is agitated, tachypneic, tachycardic with central cyanosis and a harsh systolic crescendo decrescendo murmur along the left mid-to-upper sternal border. An arterial oxygen saturation is 78% and a chest X-ray shows a boot-shaped heart. What is the best initial step in management? Administer morphine 0.2 mg/kg IM Administer propanolol 0.01 mg/kg IV Console the child in a knee-chest position Start positive pressure ventilation

Correct Answer ( C ) Explanation: This clinical scenario describes a hypercyanotic episode, also known as a 'tet spell' that can occur in children with Tetrology of Fallot (TOF). The four associated abnormalities in TOF are right ventricular hypertrophy, right ventricular outflow tract obstruction, ventricular septal defect and an over-riding aorta. Classic chest X-ray findings of TOF include a normal sized but boot-shaped heart reflecting the right ventricle with decreased pulmonary vascularity. TOF anomalies can lead to decreased pulmonary flow and right-to-left shunting through the VSD. A 'tet spell' can occur with decreased systemic vascular resistance that can result from hypotension, exertion during feeding, crying, defecation and tachycardia. This leads to increased right-to-left shunting of the blood across the VSD. The cyanotic episode is usually 15 to 30 minutes long but can lead to stroke, seizures and even death. During a spell, patients can have a right ventricular outflow tract obstruction murmur (harsh systolic crescendo decrescendo murmur along the mid-to-upper sternal border) and a VSD murmur (harsh pansystolic murmur heard best at the lower left sternal border). Management of spells aims to increase systemic vascular resistance, correct hypoxia and correct acidemia. Initial steps include administering high-flow oxygen by mask, consoling the child by cradling them in a knee-chest position, and avoiding exacerbating distress. Underlying arrhythmia, hypothermia, and hypoglycemia should also be corrected. Severe 'tet spells' may require morphine administration 0.2 mg/kg IM (A). Prolonged 'tet spells' may require IV fluids, correction of acidosis with sodium bicarbonate, beta-blockers such as propanolol (B) or esmolol, vasopressors such as phenylephrine, or even intubation or positive pressure ventilation (D).

A 43-year-old woman with a history of HIV (last CD4 count 231, viral load undetectable) presents with a sore throat that has progressively worsened over 1 week. She has been unable to tolerate solids for the last 3 days and has pain with swallowing liquids as well. Her vitals are normal. On exam, you note the findings seen in the image above. What management is indicated at this time? Acyclovir Clotrimazole troches Fluconazole Penicillin V potassium

Correct Answer ( C ) Explanation: This patient has esophagitis secondary to infection with Candida albicans. Infectious esophagitis is uncommon in immunocompetent individuals but is fairly common in immunocompromised patients (i.e. HIV/AIDS, transplant patients, chronic corticosteroid use, diabetes mellitus, chronic alcohol abuse etc.). Changes in the mucosal barrier in the esophagus in these patients leads to an increased susceptibility to infection. Candida species (specifically C. albicans) represent the most common infectious etiologies. Infections with Candida species produce characteristics white plaques. These plaques can be scraped off of mucosal surfaces revealing a raw, erythematous and sometimes bleeding base. In HIV, patients with CD4 counts <200 are more susceptible to Candida infection but this fungus can cause infections in all patients regardless of CD4 count. Although topical agents can be considered in immunocompetent patients with mild disease, patients with immunocompromise will usually require systemic medications (fluconazole, ketoconazole or itraconazole). Treatment usually runs for 3-4 weeks. Acyclovir (A) is recommended for the treatment of infectious esophagitis caused by herpes simplex virus (HSV). In HSV pharyngitis, the presenting complaints are similar to those in Candida esophagitis but examination should reveal herpetic vesicles. Clotrimazole troches (B) and nystatin are topical treatments for Candida esophagitis that can be used in immunocompetent patients with mild symptoms. Penicillin V potassium (Pen VK) (D) is the treatment of choice for pharyngitis secondary to Streptococcus pyongenes. S. pyogenes infection presents with unilateral or bilateral white tonsillar exudates.

A 34-year-old woman has been seen multiple times in the past several months for various pain-related complaints. On each occasion, no physical or laboratory findings were found to explain the symptoms. The patient is involved in a worker's compensation case and could make a significant amount of money if it is demonstrated that her physical complaints are related to work conditions. Which one of the following diagnoses characterizes her unexplained physical symptoms? Conversion disorder Factitious disorder imposed on self Malingering Somatic symptom and related disorders

Correct Answer ( C ) Explanation: This patient most likely is malingering, which is to purposefully feign physical symptoms for external gain. The most common goals of people who malinger in the emergency department are obtaining drugs and shelter. In the clinic or office, the most common goal is financial compensation. According to the DSM-5, malingering should be suspected in the presence of any combination of the following: Medicolegal presentation, marked discrepancy between the claimed distress and the objective findings, lack of cooperation during evaluation and in complying with prescribed treatment or presence of an antisocial personality disorder. Conversion disorder (A) involves a single voluntary motor or sensory dysfunction suggestive of a neurologic condition, but not conforming to any known anatomic pathways or physiologic mechanisms. Factitious disorder (B) involves adopting physical symptoms for unconscious internal gain, such as deriving comfort from taking on the role of being sick. Somatization disorder (D) is related to numerous unexplained physical symptoms that last for several years and typically begin before 30 years of age.

A 47-year-old man presents for evaluation of a whitish discoloration on his tongue for the last several days. He denies any fever or other systemic signs. On examination, the following findings are seen on the image above. You are unable to scrape the base of the plaque off with a tongue depressor. Which of the following tests is likely to be positive? Gram stain demonstrating large, ovoid, gram-positive yeast KOH prep Rapid HIV Rapid strep

Correct Answer ( C ) Explanation: This patient's tongue demonstrates the white corrugated plaques of oral hairy leukoplakia. This is a disease of the lateral tongue caused by infection of the squamous epithelium with Epstein-Barr virus. Most commonly the infection is localized to the lateral portion of the tongue, although in rare cases it may involve the floor of the mouth, palate or buccal mucosa. Clinically this may be difficult to differentiate from thrush caused by candida. *However, the lesions of oral hairy leukoplakia cannot be scraped off the surface. Although caused by Epstein-Barr, it is rarely seen in conditions other than patients with HIV infection.* A Gram stain demonstrating large, ovoid, gram-positive yeast (A) is consistent with candidiasis. A KOH prep (B) may be used to identify candida infections. On the prep, pseudohyphal elements will be seen microscopically. A rapid strep test (D) would not be positive in this individual. Strep characteristically has a white exudate over the tonsils and posterior pharynx. When using the rapid strep diagnostically, it has a high specificity, but poor sensitivity requiring follow-up culture for negative rapid tests.

A 72-year-old African-American man presents to the emergency department for disorientation. According to his family, the patient has been progressively more withdrawn, apathetic, and complaining of progressive nausea, vomiting, anorexia, drowsiness, and lethargy during the last few months. His past medical history includes type-2 diabetes controlled with diet and metformin, and an uncomplicated appendectomy 15 years ago. Vital signs are within normal range. On physical examination, he is disheveled, disoriented, drowsy, and dehydrated. Laboratory results reveal hemoglobin 13 g/dL, platelets 310,000, sodium 145, potassium 4.9, chloride 101, calcium 14.1, glucose 91, BUN 8, and creatinine 0.8. Which of the following is the most appropriate next step in management? Biphosphonates Calcitonin Hemodialysis Normal saline and furosemide

Correct Answer ( D ) Explanation: Hypercalcemia is usually symptomatic when serum calcium is higher than 12 mg/dL (normal range is 8.5 - 10.5 mg/dL). Symptoms include mental confusion, nausea, vomiting, constipation, polyuria, shortening of the QT-interval, and sometimes, cardiac arrhythmias. Levels between 15 and 18 mg/dL can lead to coma and cardiac arrest, and constitute a medical emergency. Hypercalcemia can be secondary to excessive intake or absorption, release of inordinate amounts from bone secondary to hormones, bone disease, or cancer. Primary hyperparathyroidism (e.g. hyperplasia, adenoma, or carcinoma of the parathyroid glands) and neoplasia account for 90% of hypercalcemia cases. Paraneoplastic disease may be caused by solid tumors producing PTH-like proteins, or multiple myeloma, which produces osteoclast-activating factors. Given this patient's high calcium levels, the hypercalcemia should be managed immediately, before further diagnostic studies are undertaken. *The combination of normal saline and furosemide is the treatment choice for lowering the calcium level. Note that fluid replacement should be initiated before diuretics are administered. Once intravenous fluids are initiated then diuretics such as furosemide can be started. Loop diuretics, such as furosemide, inhibit the sodium-potassium-chloride co-transporter in the thick ascending loop of Henle. The resulting loss of the electrical gradient across the membrane, promotes excretion of other electrolytes, such as calcium and magnesium, as well as fluid loss.* Dehydration and electrolyte deficits can lead to severe, even fatal, complications. Therefore, fluid and electrolyte replacement, accompanied by careful monitoring of electrolyte levels, is essential. Other side effects of loop diuretics include hyperuricemia, hyperglycemia (in rare cases leading to crises of gout or diabetes), reversible ototoxicity, serum lipid elevations, gastrointestinal disturbances, and allergic reactions. Biphosphonates (A), particularly pamidronate, alendronate, and zoledronate are used after initial hydration to inhibit bone reabsorption and maintain low serum calcium levels, especially in hypercalcemia of malignancy and Paget disease. These are used as an adjunctive treatment and should be initiated after first-line treatments are given. Other second-line options are gallium nitrate, calcitonin, and prednisone. *Calcitonin (B) lowers serum calcium in patients with multiple myeloma, carcinoma, or primary hyperparathyroidism.* Onset of action is approximately 2 h following injection, and activity lasts for 6-8 h. Emergency hemodialysis (C) can be used in life-threatening hyperkalemia and exogenous intoxication with lithium, methanol, ethylene glycol, salicylates, theophylline, and valproic acid. It is not indicated for hypercalcemia.

A 35-year-old woman presents to her primary care provider with a 3-month history of insomnia, palpitations and unintentional weight loss. A friend mentioned to her that her eyes seemed to be protruding from their sockets. Which of the following is the most appropriate diagnostic test? HbA1c level Prolactin level Random cortisol level Thyroid stimulating hormone (TSH) level

Correct Answer ( D ) Explanation: Hyperthyroidism is caused by elevated thyroid hormone (T4 or T3 or both). It can lead to a variety of symptoms including insomnia, palpitations, weight loss, heat intolerance, anxiety, fatigue, fine tremor and irregular menstrual periods. Physical exam findings include tachycardia, hyperreflexia, thinning hair, lid lag, exophthalmos, and pretibial myxedema. Hyperthyroidism may be caused by a variety of disorders including Graves disease, solitary toxic hyperactive nodule, toxic multinodular goiter, and thyroiditis. The diagnostic workup for hyperthyroidism begins with obtaining a TSH, free T4, free T3, and thyroid antibodies (Graves disease). In most cases of hyperthyroidism, the TSH is low. HbA1c (A) is used in diagnosing and monitoring blood glucose levels over time in patients with diabetes. A random cortisol level (C) is used in a patient suspected to have an adrenal abnormality such as Cushing's disease (hypercortisolism) or Addison's disease (adrenal insufficiency) while a prolactin level (B) is used to identify a patient with hyperprolactinemia likely due to a pituitary tumor or medication side effect.

A 25-year-old sexually active, homosexual man presents to the Emergency Department with a complaint of a painful right-sided inguinal lymph node. On examination, a tender and fluctuant lymph node is noted above and below the inguinal ligament. Which of the following most likely preceded the inflamed lymph node in this patient? Grouped small painful vesicles Multiple painful genital ulcers Painless ulcer lasting three weeks Painless ulcer lasting two days

Correct Answer ( D ) Explanation: Lymphogranuloma venereum (LGV) is a sexually transmitted disease more prevalent in tropical countries and is typically more prevalent in men who have sex with men. It is caused by Chlamydia trachomatis, a gram negative intracellular bacteria. Three serotypes are responsible for LGV: L1, L2, and L3. Incubation lasts for 3-21 days when a small, painless genital ulcer appears and spontaneously resolves after 2-3 days. Often the infected person is not aware of this ulcer. The secondary stage begins 7-30 days after the resolution of the primary lesion and is characterized by regional lymphadenopathy. The lymphadenopathy progresses to buboes which are painful, firm fluctuant lymph nodes which form a "groove sign" in which the adenopathy that extends above and below the inguinal ligament. Diagnosis is clinical, serologic testing for chlamydia is generally not helpful except when needed to support the diagnosis in the appropriate setting. Treatment is with doxycycline 100 mg twice a day for 3 weeks. Herpes simplex virus, type 1 or 2, initially presents as grouped small painful vesicles (A) after a viral prodrome. Typically the patient will also complain of localized tingling, burning, and pain at the site where the vesicles eventually develop. These vesicles eventually deroof and become painful ulcers. Tender lymphadenopathy may develop, but does not progress to large fluctuant nodes and will not develop above and below the inguinal ligament. Chancroid is caused by Haemophilus ducreyi, which initially forms a small pustule at the site of inoculation which progresses to multiple painful genital ulcers (B) with sharply demarcated purulent bases. A painful bubo, or large painful, fluctuant inguinal lymph node, may also develop, but a "groove sign", which is the lymph node above and below the inguinal ligament, will not develop. Syphilis, caused by the spirochete Treponema pallidum, first presents as a chancre or painless ulcer lasting three weeks (C). The chancre spontaneously heals in 3-6 weeks before the infection progresses on to secondary (characterized by a rash and lymphadenopathy) and tertiary syphilis which affects the nervous and cardiovascular system through neuropathy and widespread granulomatous lesions.

A 13-month-old boy presents to your office with fever and rash. Five days ago, he developed a fever of 103°F and had symptoms of cough, rhinorrhea, and eye redness. Yesterday, he woke up with a rash affecting his head and face that is now spreading to the rest of his body. Which of the following is most likely to confirm the diagnosis? Rapid influenza diagnostic testing Rapid strep testing Serum Epstein-Barr virus anti-viral capsid antigen Serum rubeola virus IgM antibody

Correct Answer ( D ) Explanation: Measles is a viral illness caused by the rubeola virus. It is one of the most contagious infectious diseases and is spread through contact with respiratory droplets. The virus can live for up to two hours in an area where the infected person was coughing or sneezing. Vaccine programs have helped to significantly decrease the incidence of measles, with the majority of cases being linked to international travel. In the past few years, there has been an increase in cases, mostly among unvaccinated individuals. Clinical manifestations of measles include a prodrome of high fever and "the 3 C's": cough, coryza and conjunctivitis, followed by the measles exanthem. The classic presentation of the exanthem is a maculopapular, erythematous, blanching rash that begins on the head and face and spreads downward, occurring two to four days after onset of fever. The most common laboratory test used in the diagnosis of measles is serum measles IgM antibody. Patients being evaluated for testing should be isolated and additional testing including throat or nasopharyngeal swab for viral culture and a urine sample for viral culture should be ordered. Measles is a reportable infection, and suspected cases should be reported to the local health department within 24 hours. The differential diagnosis for measles depends on whether the patient presents during the prodrome stage or after the rash has erupted. Clinical manifestations of influenza include fever and cough, although the fever seen in measles is usually higher than in influenza. Rapid influenza diagnostic testing (A) is the initial test done in suspected influenza patients. Scarlet fever is a complication of group A Streptococcus infection and presents with fever, pharyngitis, and a sandpaper-like, erythematous rash that blanches. Initial testing is with the rapid strep test (B). Patients with infectious mononucleosis present with fever, pharyngitis, fatigue, lymphadenopathy and sometimes with a generalized rash. Diagnosis is often clinical. If tested, serum Epstein-Barr virus anti-viral capsid antigen (C) is present at the onset of illness.

A 33-year-old wrestler presents with a worsening skin infection. He was seen in the ED two days prior and had an incision and drainage of an abscess on his left arm. The drainage has decreased but now he has an area of erythema that has expanded around his original abscess and a second purulent lesion on his leg. Which of the following is the most appropriate antibiotic choice? Amoxicillin-clavulanic acid Cephalexin Ciprofloxacin Trimethoprim-sulfamethoxazole

Correct Answer ( D ) Explanation: Purulent skin infections are most commonly caused by Staph aureus. Since the early 2000s, approximately 60% of these infections are from community-associated methicillin-resistant Staph aureus (MRSA). An abscess forms when bacteria replicate near the base of a hair follicle or other location beneath the epidermis. On physical examination, the area is erythematous, warm, tender, indurated and fluctuant in the center. On bedside ultrasound, the fluid cavity is easily identified as a hypoechoic space using the high-frequency linear probe. Treatment of abscesses is surgical with incision and drainage. Antibiotics are not indicated. However, in certain situations antibiotics are part of the treatment plan including: severe or extensive disease which includes multiple sites of infection, severe associated cellulitis, systemic illness, immunocompromise, locations where drainage is not possible, or failure to respond to medical treatment. When antibiotics are prescribed, coverage against community-associated MRSA is mandatory. Trimethoprim-sulfamethoxazole has widespread activity against the organism in the US. Amoxicillin-clavulanic acid (A) is a beta-lactam antibiotic used for many infections, including Staph aureus. However, it has no activity against the MRSA strain. Cephalexin (B) is a cephalosporin with adequate Staph coverage, however, it is ineffective against MRSA. Ciprofloxacin (C) is a quinolone not considered first-line in the treatment of skin and soft tissue infections. It also is ineffective against MRSA.

A two-month-old boy is in the clinic for a well child exam. He has been exclusively breastfeeding and has been gaining weight appropriately. He has been staying asleep longer for four to five hours. His stools are soft. He is also less fussy. His length is at the 90th percentile and weight is at the 85th percentile. Physical examination is normal. He is due for his vaccinations, and the mother gives her consent. Which of the following is the best statement regarding the rotavirus vaccine? Dietary restriction is needed before or after rotavirus vaccine is administered Rotavirus vaccines are administered intramuscularly Rotavirus vaccines cannot be given at the same visit as the other routine immunizations Vaccine doses that are regurgitated, spit out, or vomited do not need to be repeated

Correct Answer ( D ) Explanation: Rotavirus was the most common cause of severe gastroenteritis in infants and children before mass immunization. Thus, universal immunization of infants against rotavirus is recommended by the Centers for Disease Control and Prevention, the World Health Organization, the American Academy of Pediatrics. The recommended routine schedules for pentavalent human-bovine rotavirus reassortant vaccine (RV5) and attenuated human rotavirus vaccine (RV1) differ. RV5 is administered in three oral doses at two, four, and six months of age while RV1 is administered in two oral doses at two and four months. Rotavirus vaccines are administered orally. Dietary restriction, including breastfeeding, is not needed before or after rotavirus vaccine is administered. Doses that are regurgitated, spit out, or vomited need not be repeated. Multiple randomized trials have demonstrated that breastfeeding does not affect rotavirus vaccine efficacy. Dietary restriction is needed before or after rotavirus vaccine is administered (A) is false since dietary restriction is not needed. Rotavirus vaccines are administered intramuscularly (B) is wrong because it is given orally. Rotavirus vaccines cannot be given at the same visit as the other routine immunizations (C) is false because it can be given at the same visit along with the other immunizations.

A 27-year-old woman presents to your office with complaints of pain and discomfort. She tells you that she has seen numerous doctors and none of them have been able to help her. Her symptoms today include nausea, gluten sensitivity, irregular menses, weakness in her legs, headache, dysuria, dyspareunia, and back pain. She would like you to do a CT scan to determine the cause of her complaints. Which of the following is the most likely diagnosis? Chronic pain syndrome Factitious disorder Munchausen's syndrome Somatization disorder

Correct Answer ( D ) Explanation: Somatization disorder is characterized by physical complaints from various organ systems. Diagnostic criteria include a history of multiple physical complaints starting prior to age 30 years resulting in the patient seeking treatment from many different medical providers. The physical complaints cause significant impairment in the patient's life, including occupational and social functioning. Symptoms must consist of four pain symptoms, two gastrointestinal symptoms, one sexual symptom and one pseudoneurologic symptom not fully explained by a confirmed medical condition. Impulsive and demanding behavior, such as requesting specific diagnostic testing, may occur with patients who meet criteria for this disorder, however there is no evidence that the patient is being deceptive. Treatment focuses on behavior modification including regularly scheduled, brief visits to the medical provider without a diagnostic focus. Chronic pain syndrome (A) is defined as pain lasting longer than 3 months. While patients may seek treatment from numerous medical providers, the criteria for diagnosis does not require the same number of complaints that somatization disorder requires. Factitious disorder (B) and Munchausen's syndrome (C) both involve purposefully feigned illnesses on the spectrum of mild to severe. Individuals with factitious disorder may exaggerate physical symptoms or mimic illness. Munchausen's syndrome is a severe form of factitious disorder that may involve the patient traveling extensively to seek medical care, requesting multiple invasive procedures, and even impersonating another individual in order to obtain treatment. The intention of the patient to be in the sick role and the use of deception to attain this goal are what differentiates factitious disorder and Munchausen's syndrome from somatization disorder. UPDATE: The Diagnostic and Statistical Manual of Mental Disorders, Fifth Edition (DSM-5) does not use the term somatization, and has eliminated the category of diagnoses called somatoform disorders. For patients with prominent somatic symptoms that cause distress and impair psychosocial functioning, DSM-5 has replaced the category of somatoform disorders with a category called somatic symptom and related disorders.

A 15-year-old girl is seen in your office for a routine visit. Her mother has type 2 diabetes mellitus. On physical exam, she is noted to have a body mass index of 34 and acanthosis nigricans on her neck. You check a hemoglobin A1c, and it is 6.8 percent. Which of the following medications is appropriate to prescribe? Glyburide Levothyroxine Lisinopril Metformin

Correct Answer ( D ) Explanation: The girl described in this vignette is obese and has a family history of type 2 diabetes mellitus. The acanthosis nigricans present on her neck along with an elevated hemoglobin A1c is consistent with a diagnosis of diabetes. Current recommendations indicate that she should be aggressively counseled on lifestyle modifications and begin pharmacologic therapy. Metformin is the only drug approved by the FDA for the treatment of diabetes mellitus type 2 in children older than 10 years of age. It is usually the first drug given when starting treatment and acts by decreasing hepatic glucose production (gluconeogenesis) and decreasing insulin resistance. Metformin is generally tolerated well but can cause diarrhea, nausea, and abdominal pain. It should be discontinued prior to receiving any IV contrast due to the risk of lactic acidosis. Glyburide (A) is a sulfonyluruea that works by directly increasing insulin secretion. It is usually not used in the pediatric population as first-line treatment for type 2 diabetes. Levothryoxine (B) is used to treat cases of thyroid dysfunction, resulting in decreased thyroid hormone production. Lisinopril (C) is an angiotensin-converting enzyme inhibitor that may be required in this patient if she develops hypertension or shows signs of diabetic nephropathy such as proteinuria.

A 16-year-old boy is in clinic for a routine health check. He is doing well in school and has plans to join the military when he reaches 18 years of age. He practices and plays basketball about 60 minutes per day. He denies smoking and drug use. Physical examination is normal. At this visit, you recommend meningococcal vaccine booster. Which of the following is the best statement regarding meningococcal infection? Chemoprophylaxis can be administered two weeks after exposure to the index case Droplet precaution is not necessary for patients infected with meningococcal meningitis Nasopharyngeal cultures are needed in determining the need for chemoprophylaxis Prophylaxis is indicated in close contacts of patients with meningococcal infection

Correct Answer ( D ) Explanation: The methods for the prevention of meningococcal infection include antimicrobial chemoprophylaxis following identification of an index case, use of droplet precautions, vaccination prior to exposure, and avoidance of risk factors. Antimicrobial chemoprophylaxis was first used successfully to abort the spread of meningococcal infection in the 1930s. Prophylaxis is indicated in close contacts of patients with meningococcal infection and should be given as early as possible following the exposure. Close contacts may include individuals exposed in the following way: household members, roommates, intimate contacts, contacts at a childcare center, young adults exposed in dormitories, military recruits exposed in training centers. Also included are individuals who have been exposed to oral secretions. Because the rate of secondary disease for close contacts is highest immediately following onset of disease in the index patient, antimicrobial chemoprophylaxis should be administered as early as possible (ideally less than 24 hours after identification of the index patient). Chemoprophylaxis can be administered two weeks after exposure to the index case (A) is wrong. This is because chemoprophylaxis administered 14 days after exposure to the index case is probably of limited or no value and is therefore not recommended by the United States Centers for Disease Control and Prevention (CDC). The statement that droplet precaution is not necessary for patients infected with meningococcal meningitis (B) is false. In fact, droplet precautions should be continued for 24 hours after institution of effective antibiotics in patients with suspected or confirmed Neisseria meningitidis infection. Nasopharyngeal cultures are needed in determining the need for chemoprophylaxis (C) is wrong since it is not necessary to obtain nasopharyngeal cultures before starting chemoprophylaxis.

A 10-year-old boy presents to the emergency department complaining of nausea, vomiting, perioral numbness, blurry vision, and feeling his tactile sensation of hot temperature seems off. He admits to eating out and having grouper at a local restaurant. No other family members report illness. Which of the following is the most appropriate treatment? Activated charcoal Antihistamine Hemodialysis Mannitol

Correct Answer ( D ) Explanation: The poisoning is most consistent with Ciguatera toxicity. Ciguatera poisoning is endemic to fish in the south Pacific Ocean and less so in the lower Atlantic states. Since these fish are exported around the world, the toxicity can occur anywhere. Ciguatoxin concentrates in larger fish (barracuda, grouper, red snapper, and parrot fish) that eat smaller fish. The small fish feed in coral reef dinoflagellates that form the toxin (Gambierdiscus toxicus). The toxin is odorless, colorless, and tasteless and is not deactivated by heat. Symptoms begin two to 30 hours after ingestion and are associated with gastrointestinal findings (abdominal pain, vomiting, and diarrhea), unique neurological findings (hot-cold reversal, sensation of tooth looseness, ataxia, and altered mental status), and cardiovascular findings (bradycardia, hypotension, pulmonary edema). Management is mainly supportive with antiemetics, IV fluids for hypotension and atropine for bradydysrhythmia. Mannitol is administered to prevent or alleviate neurologic morbidity. Activated charcoal (A) has no role as this toxin is not absorbed and only small amounts are necessary to cause neurologic sequela. Antihistamines (B) are used in the food-borne toxin associated with scombroid poisoning. Scombroid, unlike ciguatera, presents with flushing, redness, headache and metallic or pepper taste in mouth. It is caused by different types of fish than those associated with Ciguatera toxicity. Hemodialysis (C) has no role in this toxic ingestion.

A 34-year-old man, with no past medical history, presents to the emergency department complaining of burning epigastric pain radiating to the back for the last 3 months. The pain started a couple of hours after eating and occurs at night. The patient does not take any medications. Vitals are T 37°C, HR 82, BP 138/62 RR 18, and oxygen saturation 100% on room air. Physical examination is significant for brown, heme-positive stool. Once the diagnosis is confirmed, which of the following is the most appropriate to treat this condition? Antacids Aspirin Carafate Clarithromycin, amoxicillin, and a proton-pump inhibitor (PPI)

Correct Answer ( D ) Explanation: This patient presents with symptoms suggestive of peptic ulcer disease (PUD). PUD is the collective term for gastric and duodenal ulcers. PUD is a common disease effecting 4 million patients in the U.S. each year. The most common etiologic agent of PUD is infection with Helicobacter pylori and eradication of the bacteria leads to more rapid ulcer healing, prevents relapse and decreases the rate of complications. The second most common cause of PUD is the use of non-steroidal anti-inflammatory drugs (NSAIDs). About 1% of patients develop PUD secondary to increased levels of gastrin from gastrin-secreting tumors. Patients typically present with epigastric abdominal pain which occurs 2-5 hours after eating or when sleeping. Pain is often relieved after eating or with antacids. Complications include perforation, upper gastrointestinal hemorrhage and gastric outlet obstruction. Initial treatment of presumptive PUD (definitive diagnosis can only be made on endoscopy) includes lifestyle changes including smoking cessation, reduction or elimination of alcohol use, and stopping NSAID and aspirin use. If NSAIDs are not being used by a patient with suspected PUD (as in this case), the current recommendation is to test for H. pylori with antibody detection, urea breath test or fecal antigen tests. This is often done on an outpatient basis with a gastroenterologist or primary care provider. Once H.pylori is confirmed as the cause, triple therapy, which consists of clarithromycin 500 mg twice a day, amoxicillin 1 gram twice a day (or metronidazole 500 mg twice a day if penicillin allergic) and a PPI once a day for 10-14 days, should be initiated. Aspirin (A) ​will exacerbate ulcers, not treat them. Antacids (B) (e.g. bismuth subsalicylate, magnesium hydroxide) can give symptomatic relief of dyspepsia but do not treat the underlying cause of PUD and may delay definitive treatment by reducing symptoms. Carafate (C) can provide symptomatic care but will not eradicate H. pylori.

A 64-year old man presents with right-sided abdominal pain and the following rash. Which of the following is the most likely diagnosis? Coxsackie A16 Herpes simplex type 1 Rubeola Varicella zoster

Correct Answer ( D ) Explanation: Varicella Zoster is the cause of both varicella (chickenpox) and zoster (shingles). Chickenpox is generally an infection of childhood that is characterized by a vesicular rash in crops of varying stages. The transmission is via respiratory droplets and contact with ruptured vesicles. Shingles is a reactivation of the latent virus from the dorsal root ganglia. Shingles can occur at all ages but is most commonly seen in patients older than 50 years. Shingles presents in a unilateral dermatomal distribution of vesicles. It is associated with pain and burning that may precede the eruption by up to 72 hours. The rash evolves quickly from an erythematous, maculopapular eruption to a cluster of vesicles. The duration of illness is usually 7-10 days. Transmission is by contact with the ruptured vesicles and via respiratory droplets. Treatment consists of oral anti-viral medications (acyclovir or valacyclovir), aluminum acetate soaks and pain control. Vaccines are available for both chickenpox and shingles. The shingles vaccine is indicated in patients older than 60 years, and in some patients older than 50 years. Coxsackie (A) commonly known as hand-foot-mouth disease. It is a highly contagious viral entity commonly seen in children. Lesions are ulcerative on the oral mucosa, palms and soles. Epidemics are common in the summer months when children are playing together in public pools. Herpes simplex type 1 (B) is a virus capable of producing a variety of infections of the skin, genitalia, and CNS. Primary infections have a flu-like prodrome. HSV 1 infection usually occurs on the oral mucosa. It can present as a cluster of vesicles on, above, or below the lips and can be preceded by tingling in the area of reactivation. Rubeola (measles) (C) is a viral illness characterized by a prodrome of fever, conjunctivitis, and coryza followed by a generalized maculopapular rash that begins behind the ears and descends down the body.

A 20-year-old woman fainted while standing in line at the grocery store. The patient admits to feeling nauseated and diaphoretic before the episode. She denies bowel or bladder incontinence and did not experience a post ictal state. The woman in line behind her observed jerking motions of her face and fingers. She has no past medical history and does not take any medications. What is the most likely diagnosis? Hypertrophic cardiomyopathy Orthostatic hypotension Seizure Vasovagal syncope

Correct Answer ( D ) Explanation: Vasovagal, also called neurocardiogenic syncope, is often called the "common faint" and is the most common cause of syncope. Vasovagal syncope is a transient loss of consciousness caused by systemic hypotension and cerebral hypoperfusion. It is a neurally mediated reflex response characterized by bradycardia or peripheral vasodilation. Patients with vasovagal syncope are usually young and healthy. The clinical presentation of "classic" vasovagal refers to syncope triggered by provoking factors such as noxious stimuli, pain, blood draw, intense emotion, fear of bodily injury, prolonged standing, or heat exposure. Prodromal symptoms include feeling warm, sweating, nausea, and pallor. In addition, some patients present with myoclonic or other involuntary movements that are suggestive of a seizure but are actually due to cerebral hypoxia secondary to hypotension. This seizure like activity, historically, has been referred to as a "fainting fit." Vasovagal syncope is not dangerous; however, it is important that other more serious causes of syncope be ruled out. Hypertrophic cardiomyopathy (A) is also a cause of syncope in young and heathy individuals. However, it is usually related to exercise and associated with angina secondary to an exertion induced left ventricular outflow tract obstruction. Orthostatic hypotension (B) causes syncope upon assuming an upright position from supine or sitting. It is often caused by hypovolemia, medications or autonomic nervous system disorders. Seizure (C) causes a loss of consciousness, but it is associated with tonic-clonic jerking of extremities. Additionally, seizures often cause bowel or bladder incontinence as well as a post-ictal state. Syncopal events are not associated with incontinence and usually have a rapid and complete recovery.

Which of the following is associated with a shortened PR interval? Brugada Syndrome Mobitz 2° AV Block Wellens Syndrome Wolff-Parkinson-White syndrome

Correct Answer ( D ) Explanation: Wolff-Parkinson-White syndrome (WPW) is a congenital condition in which there is an abnormal accessory conduction pathway called the bundle of Kent. This accessory conduction pathway causes a premature depolarization of the ventricles by bypassing the AV node. The AV node typically acts to delay depolarization (the PR interval) to the ventricles to allow them to fill before contracting (QRS complex). The accessory pathway causes early depolarization of the ventricles resulting in an early QRS complex, thereby shortening the PR interval. On an ECG, this is represented by an upslope of the QRS complex (delta wave) that causes a widened QRS complex (>120 msec). The primary significance of WPW syndrome is that it predisposes patients to the development of tachydysrhythmias, particularly atrial fibrillation. Rapid Review Wolff-Parkinson-White Syndrome PR interval <120 msec, delta wave, wide QRS complex Accessory pathway, bundle of Kent Predisposed to tachydysrhythmias Procainamide Brugada Syndrome (A) is a hereditary condition characterized by a right bundle branch block-like pattern with ST elevation in Leads V1 to V3. Wellens Syndrome (C) is described by a large inverted T wave in Leads V2 and V3 and is associated with critical stenosis of the proximal left anterior descending coronary artery. Both Brugada and Wellens syndromes have normal PR intervals. Mobitz 2° AV Block (B) occurs at the level of AV node. Depolarizations from the atria are blocked and do not reach the ventricles, resulting in some P waves not being followed by a QRS complex (thereby, having no PR interval). The P waves that are followed by QRS complexes have normal PR intervals.

Question: What is the diagnostic murmur auscultated in aortic stenosis?

Crescendo-decrescendo systolic murmur. Rapid Review Aortic Stenosis Patient will be older With a history of diabetes, hypertension Complaining of dyspnea, chest pain, syncope PE will show crescendo-decrescendo systolic murmur that radiates to the carotids, paradoxically split S2, S4 gallop Most commonly caused by degenerative calcification Treatment is aortic valve replacement Comments: murmur decreases with valsalva

Question: Which antibiotic should be used if macrolide resistance is suspected or documented in a patient with atypical pneumonia?

Doxycycline or a fluoroquinolone. Rapid Review Atypical Pneumonia Patient will be complaining of gradual onset of dry cough, dyspnea, and extra-pulmonary symptoms such as headache, myalgias, fatigue, and GI disturbance PE will show rales with auscultation of lung fields Most commonly caused by Mycoplasma pneumoniae Treatment is azithromycin

Question: What is the name of the ECG finding classically associated with cardiac tamponade?

Electrical alternans. Rapid Review Acute Pericarditis Idiopathic > viral (Coxsackie) Pleuritic chest pain radiating to the back Pain ↓ with leaning forward PE: pericardial friction rub ECG: diffuse ST elevation, PR depression NSAIDs

Question: What is the quickest way to diagnose acute angle glaucoma?

Elevated intraocular pressure using tonometry. Rapid Review Acute Angle-Closure Glaucoma Patient will be entering a dark room or movie theater Complaining of acute unilateral painful vision loss, vomiting, and seeing halos around lights PE will show cloudy cornea and fixed mid-dilated pupil Labs will show ↑ IOP ( > 21 mm Hg) Treatment is topical ßBs, carbonic anhydrase inhibitors, steroids, miotics

Question: What diagnostic studies are used in women with symptoms of vaginitis?

Examination of vaginal discharge for fishy odor and pH along with microscopy. Rapid Review Vulvovaginal Candidiasis Patient with a history of diabetes, HIV, recent abx use Complaining of pruritus, dysuria, dyspareunia PE will show white, cottage cheese-like discharge Labs will show pH < 4.5, pseudohyphae, spores Most commonly caused by Candida Treatment is fluconazole

A woman presents with reduced exercise tolerance and dyspnea on exertion. On exam, you hear the murmur heard in the audio clip above. Which of the following is the correct diagnosis? Aortic regurgitation Aortic stenosis Mitral regurgitation Mitral stenosis

Explanation: The murmur of mitral stenosis includes a loud S1 and an opening snap in early diastole accompanied by a low-pitched, rumbling diastolic apical murmur. Mitral valve stenosis most commonly results from rheumatic heart disease. Other causes include atrial myxoma, congenital abnormalities, and calcific valve degeneration. The murmur of mitral stenosis occurs from mitral valve narrowing leading to increasing pressures across the mitral valve with subsequent left atrial hypertrophy and eventual dilation with left heart failure. The murmur of aortic regurgitation (A) is a blowing diastolic murmur heard best at the sternal border. The murmur of aortic stenosis (B) is a crescendo-decrescendo systolic murmur that radiates to the neck. The murmur of mitral regurgitation (C) is a loud holosystolic murmur heard best at the apex and radiates to the base.

Question: What is the DSM-5 term for the formerly described "Munchausen syndrome by proxy"?

Factitious disorder imposed on another. Rapid Review Somatoform Disorders Genuine sx with no identifiable cause F > M Conversion disorder: sudden loss of sensory or motor function Hypochondriasis: Preoccupation with having illness Pain disorder Somatization disorder: many complaints in ≥ 1 organ systems Rx: Reassurance, minimize providers

Question: Which patients with suspected renal stone require imaging?

First episode of renal colic, unclear diagnosis, concomitant UTI, the elderly. Rapid Review Nephrolithiasis Patient will be complaining of flank pain radiating to groin PE will show a patient that won't lay still and hematuria Diagnosis is made by helical CT Most commonly caused by calcium oxalate Struvite: staghorn calculi, urease producing bacteria Uric acid: radiolUcent on xray, gout Cystine: children with metabolic diseases Most common location is the ureterovesiclular junction (UVJ) Treatment is: < 5 mm: likely to pass spontaneously > 8 mm: unlikely to pass, lithotripsy

Question: Diagnostic surveillance testing for bladder cancer is largely determined by the results of which cytologic test?

Fluorescence in situ hybridization (FISH). Rapid Review Bladder Cancer Patient will be older With a history of smoking Complaining of painless hematuria Diagnosis is made by cystoscopy Most common type is transitional cell carcinoma

Question: In addition to the TSH level, what additional testing should be done in the workup of hyperthyroidism?

Free T4, T3, and thyroid antibodies. Rapid Review Hyperthyroidism Patient will be complaining of heat intolerance, palpitations, weightloss, tachycardia, and anxiety PE will show hyperreflexia Labs will show low TSH and high free T4 Most commonly caused by Graves disease (autoimmune against TSH receptor) Treatment is methimazole or PTU Comments: Propylthiouracil (PTU) P for pregnant

Question: Which cosmetic side effect may result from steroid injection of a chalazion?

Hypopigmentation. Rapid Review Chalazion Patient will be complaining of a painless mass on the upper eyelid Most commonly caused by obstruction of Meimobian gland Treatment is warm compresses, topical antibiotics

Question: List some common side effects of IV labetalol?

Hypotension (as with other IV antihypertensives), dizziness, paresthesias (especially of the scalp) and bronchospasm. Rapid Review Hypertensive Emergency Hypertension with acute end-organ system injury Encephalopathy, cardiac ischemia, renal ischemia Objectives: reduce MAP 25% in first hour, normalize BP over the next 8 to 24 hours Reduction of MAP > 25% may cause end-organ ischemia IV antihypertensives (labetalol or nicardipine)

Question: Ascariasis is commonly found is which part of the gastrointestinal tract?

Ileum. Rapid Review Ascariasis: Patient with a history of travel Complaining of cough, wheezing, abdominal pain, and diarrhea Diagnosis is made by stool studies Most commonly caused by Ascaris lumbricoides round worm Treatment is albendazole

Question: What is the mechanism of action of thiazolidinediones, pioglitazone, and rosiglitazone?

Increase insulin sensitivity in muscle and fat. Rapid Review Acanthosis Nigricans: Patient will be obese or diabetic PE will show thickened, velvety, darkly pigmented plaques on the neck or axillae Comments: Screen for diabetes in those not yet diagnosed

Question: Which chromosome abnormality is Tetralogy of Fallot associated with?

It is associated with chromosome 22 deletions and DiGeorge syndrome. Rapid Review Tetralogy of Fallot Patient with a history of episodes of cyanosis (tet spells) and squatting for relief PE will show pulmonic stenosis, right ventricular hypertrophy, overriding aorta, VSD CXR will show "bootshaped" heart Comments: Most common cyanotic congenital heart disease Mnemonic: PROVe:: Pulmonic stenosis, Right ventricular hypertrophy, Overriding aorta, VSD

Question: What is the name of the oral lesions commonly seen with measles?

Koplik's spots. Rapid Review Measles (Rubeola): Patient will be an unvaccinated young child With a history of a maculopapular rash that started on head and spreads toward feet Complaining of high fever, cough, conjunctivitis, coryza PE will show red spots with blue/white center on buccal mucosa (Koplik spots) Diagnosis is made clinically Treatment is supportive care

Question: What are the most common sites of hemorrhage in massive hemoptysis?

Massive hemoptysis typically involves the bronchial or pulmonary arteries. Rapid Review Acute Bronchitis Viruses > bacteria Most common cause of minor hemoptysis Hallmark: cough (usually productive), <1week Symptomatic treatment, bronchodilators Routine ABX therapy not indicated

Question: What is the prognosis for polycythemia vera?

Median survival from treatment to death is 9-14 years. Rapid Review Polycythemia Vera ↑ RBCs, ↑ WBCs, ↑ PLTs HCT > 60%, ↓ EPO Pruritis after shower Platelet dysfunction Sludging of blood, hyperviscosity Death: thrombosis, hemorrhage Phlebotomy, hydroxyurea, aspirin

Question: What is the most common complication of Boerhaave syndrome?

Mediastinitis. Rapid Review Boerhaave Syndrome Full-thickness esophageal rupture Most commonly occurs at the left posterolateral distal esophagus Severe chest pain Hamman's crunch Mediastinitis Water soluble oral contrast for EGD

Question: Name the condition of lateral femoral cutaneous nerve entrapment under the inguinal ligament.

Meralgia paresthetica. Rapid Review Meralgia Paresthetica Lateral femoral cutaneous neuropathy Inguinal ligament injury Dysesthesia/numbness of proximal anterolateral thigh Rx: loose clothing, weight loss

Question: What is the gold standard for diagnosing myocarditis?

Myocardial biopsy. Rapid Review Dressler's Syndrome Patient with a history of MI Complaining of pleuritic chest pain radiating to the back, worse when laying back, improved when leaning forward PE will show tachycardia and pericardial friction rub ECG will show PR depression, PR elevation (aVR), diffuse ST segment elevation (concave) Treatment is colchicine, steroids, NSAIDs (less often)

Question: Does the presence of Prehn's sign rule out testicular torsion?

No. Rapid Review Epididymitis Patient will be complaining of gradual onset unilateral scrotal pain PE will show increased color flow on doppler, relief with testicular elevation (Prehn's sign) Most commonly caused by < 35 yo: C. trachomatis, N. gonorrhea > 35 yo: E. coli, Pseudomonas Treatment is < 35 y/o ceftriaxone/doxycycline, > 35 y/o ciprofloxacin

Question: What is the definition of odynophagia and dysphagia?

Odynophagia means pain on swallowing, dysphagia means difficulty swallowing. Rapid Review Infectious Esophagitis Increased in immunosuppression Odynophagia/dysphagia, chest pain Topical antifungal (immunocompetent) Oral antifungals (immunompromised)

Question: Which exam is recommended every 6-12 months in those on hydroxychloroquine?

Ophthalmologic exam, due to this medication's potential adverse events of retinal deposition and retinopathy. Rapid Review Rheumatoid Arthritis Women in 20s/30s AI destruction of synovial joints Morning stiffness lasting > 30 minutes MCP, PIP Bilateral ulnar deviation at MCP Baker's cyst Systemic sx Lung: interstitial fibrosis, effusions Spine: atlantoaxial joint subluxation; cord/vertebral artery compression Anemia NSAIDs

Question: Which complication of sinusitis is characterized by edema and swelling of the forehead?

Osteomyelitis of the frontal bone (Pott puffy tumor). Rapid Review Sinusitis Patient will be complaining of pain over sinuses PE will show purulent rhinorrhea Most commonly caused by viral URI Treatment is supportive care Comments: bacterial sinusitis - persistent symptoms for more than 10-14 days - amoxicillin-clavulanate

Question: In patients with appendicitis, what physical sign occurs when right lower quadrant pain is elicited by passive right hip extension?

Psoas Sign. Rapid Review Appendicitis Patient will be complaining of fever, pain that began periumbilical then moved to RLQ, nausea and anorexia PE will show Psoas sign (RLQ pain on extension of right hip), Obturator sign (RLQ pain on internal rotation of flexed right hip), Rovsing sign (right lower quadrant pain when the left lower quadrant is palpated) Diagnosis is made by ultrasound, CT Most commonly caused by fecalith Treatment is surgery

Question: What is the passive immunization to rabies?

Rabies immunoglobulin. Rapid Review Rabies Patient with a history of exposure to raccoons, bats, or skunks Complaining of hydrophobia, agitation, spasms Treatment is wound care (scrubbing), Ig at wound site, vaccination

Question: What is the most commonly associated pathogen with hordeolum formation?

Staph. aureus. Rapid Review Chalazion Patient will be complaining of a painless mass on the upper eyelid Most commonly caused by obstruction of Meimobian gland Treatment is warm compresses, topical antibiotics

Question: Which bacteria is responsible for the majority of food-borne toxicity in the United States?

Staphylococcus. Rapid Review Ciguatera Poisoning: Second most common fish-borne toxin Cause by dinoflagellate toxin Barracuda, grouper, red snapper Hot-cold reversal Supportive care, atropine, mannitol

Question: Most testicular cancers are uniform and smooth except for which irregular-border subtype?

Teratoma. Rapid Review Testicular Cancer 15-35 yo Most common risk factor: cryptorchidism Most common tumor: seminoma Unilateral painless testicular mass Pre-aortic node metastases Markers: hCG, AFP

Question: In which organ is 90% of the body's vitamin A stored?

The liver. Rapid Review Vitamin A Deficiency Patient will be complaining of dry eyes and difficulty seeing at night PE will show Bitot spots on the conjunctiva

Question: What is the significance of the Tarasoff vs. Regents case?

This case establishes the physicians duty to breach patient confidentiality if the patient poses a significant threat to a 3rd party. Rapid Review Child Abuse Bucket handle fracture Posterior rib fractures Fractures of different ages Cutaneous: bruises, bites, burns Shaken baby syndrome: retinal hemorrhages

Question: What other opportunistic infection can be prevented with TMP-SMX?

Toxoplasma gondii encephalitis.

Question: Is eosinophilia seen in patients with enterobiasis?

Unlike most other helminthic infections, eosinophilia is not seen in enterobiasis. Rapid Review Pinworms Patient will be a child Complaining of nocturnal pruritus ani Diagnosis is made by tape test Most commonly caused by ingestion of Enterobius vermicularis eggs Treatment is mebendazole

Question: With any complaint of urinary incontinence, it is important to first rule out which condition?

Urinary tract infection. Rapid Review Overflow Incontinence Detrusor hyporeflexia or outflow obstruction → constantly full bladder → dribbling Dribbling, low urine flow Rx: α-blockers

Question: What is antidromic conduction?

When the impulse is conducted anterograde through the accessory pathway and retrograde through the AV node. The QRS complex is wide.

Question: What is the treatment for primary hypoparathyroidism?

While PTH analogues are used to treat osteoporosis, they are not FDA approved for hypoparathyroidism. As such, calcium and vitamin D are the mainstays of treatment. Rapid Review Hypoparathyroidism Causes: autoimmune > damage during thyroid surgery ↓ Ca, ↑ phosphorus Hypocalcemia sx


Related study sets

MKT-001- Marketing.... what is it?

View Set

chapter 1- section 1.4 mini notes

View Set

Biology - Genetics - 3.6 - Punnett Squares

View Set